Что дает минус на плюс в математике: Почему минус на минус всегда даёт плюс? – статья – Корпорация Российский учебник (издательство Дрофа – Вентана)

Содержание

Как понять, почему «плюс» на «минус» дает «минус» ?

Как понять, почему «плюс» на «минус» дает «минус» ?

Слушая учителя математики, большинство учеников воспринимают материал как аксиому. При этом мало кто пытается копнуть глубже и понять, почему «минус» на «плюс» дает знак «минус», а при умножении двух отрицательных чисел получается положительное.

Содержание

  • 1 Законы математики
  • 2 Аксиома кольца
  • 3 Выведение аксиом для отрицательных чисел
  • 4 Умножение и деление двух чисел со знаком «-»
  • 5 Общие математические правила

Законы математики

Большинство взрослых не могут объяснить себе или своим детям, почему это так. Они прочно усвоили этот материал в школе, но даже не пытались понять, откуда взялись эти правила. Но тщетно. Часто современные дети не так уверены в себе, они должны докопаться до сути вопроса и понять, например, почему «больше» за «меньше» дает «меньше». А иногда сорванцы специально задают сложные вопросы, чтобы насладиться моментом, когда взрослые не могут дать внятного ответа. А если молодой учитель попадет в беду, это настоящая катастрофа…

Кстати, следует отметить, что приведенное выше правило действует как для умножения, так и для деления. Произведение отрицательного и положительного числа даст только «минус». Если мы говорим о двух цифрах со знаком «-», результатом будет положительное число. То же самое и с делением. Если одно из чисел отрицательное, частное также будет со знаком «-».

Чтобы объяснить правильность этого закона математики, необходимо сформулировать аксиомы кольца. Но для начала нужно понять, что это такое. В математике кольцом принято называть множество, в котором задействованы две операции с двумя элементами. Но лучше всего подойти к этому на примере.

Аксиома кольца

Есть несколько математических законов.

  • Первый из них является смещаемым, по его словам, C + V = V + C.
  • Второй называется комбинацией (V + C) + D = V + (C + D).

Они также подчиняются умножению (V x C) x D = V x (C x D).

Никто не отменял правила, согласно которым открываются круглые скобки (V + C) x D = V x D + C x D, также верно, что C x (V + D) = C x V + C x D.

Кроме того, было обнаружено, что в кольцо может быть введен специальный нейтральный элемент сложения, использование которого будет истинным: C + 0 = C. Кроме того, для каждого C существует противоположный элемент, который можно обозначить как (- C). В этом случае C + (-C) = 0.

Выведение аксиом для отрицательных чисел

Приняв вышеуказанные утверждения, можно ответить на вопрос: «Какой знак« плюс »у« минуса »?» Зная аксиому об умножении отрицательных чисел, необходимо подтвердить, что действительно (-C) x V = — (C x V). А также, что верно следующее равенство: (- (- C)) = C.

Для этого вам сначала нужно будет доказать, что у каждого из элементов есть только один противоположный «брат». Рассмотрим следующий демонстрационный пример. Попробуем представить, что для C два числа противоположны — V и D. Отсюда следует, что C + V = 0 и C + D = 0, т.е. C + V = 0 = C + D. Помня о законах смещения и о свойствах числа 0 можно считать сумму всех трех чисел: C, V и D. Попробуем вычислить значение V. Логично, что V = V + 0 = V + (C + D) = V + C + D, потому что значение C + D, как принято выше, равно 0. Следовательно, V = V + C + D.

Аналогично отображается значение D: D = V + C + D = (V + C) + D = 0 + D = D. Исходя из этого, становится ясно, что V = D.

Однако, чтобы понять, почему «больше» за «меньше» дает «меньше», необходимо понимать следующее. Итак, для элемента (-C), C и (- (- C)) противоположны, то есть они равны друг другу.

Тогда очевидно, что 0 x V = (C + (-C)) x V = C x V + (-C) x V. Отсюда следует, что C x V противоположен (-) C x V, поэтому (- С) х V = — (С х V).

Для полной математической строгости также необходимо подтвердить, что 0 x V = 0 для любого элемента. Если следовать логике, то 0 x V = (0 + 0) x V = 0 x V + 0 x V. Это означает, что добавление продукта 0 x V никак не меняет установленное количество. Ведь этот товар нулевой.

Зная все эти аксиомы, можно вывести не только, сколько «больше» дает «меньше», но также и то, что получается при умножении отрицательных чисел.

Умножение и деление двух чисел со знаком «-»

Если не вникать в математические нюансы, можно попробовать более простой способ объяснить правила действий с отрицательными числами.

Предположим, что C — (-V) = D, согласно этому C = D + (-V), то есть C = D — V. Переносим V и получаем, что C + V = D. То есть C + V = C — (-V). Этот пример объясняет, почему в выражении, в котором подряд идут два «минуса», указанные знаки следует заменить на «плюс». Теперь займемся умножением.

(-C) x (-V) = D, вы можете добавлять и вычитать два идентичных продукта к выражению, что не изменит его значение: (-C) x (-V) + (C x V) — (C x V) = D.

Вспоминая правила работы со скобками, получаем:

1) (-C) x (-V) + (C x V) + (-C) x V = D;

2) (-C) x ((-V) + V) + C x V = D;

3) (-C) x 0 + C x V = D;

4) C x V = D.

Отсюда следует, что C x V = (-C) x (-V).

Точно так же вы можете доказать, что разделение двух отрицательных чисел приведет к положительному.

Общие математические правила

Конечно, такое объяснение не сработает для учеников начальной школы, которые только начинают учить абстрактные отрицательные числа. Им лучше всего объяснять видимые объекты, манипулируя знакомым термином через зеркало. Например, там есть придуманные, но несуществующие игрушки. Они могут отображаться со знаком «-». Умножение двух зеркальных объектов переносит их в другой мир, который приравнивается к настоящему, то есть в результате мы получаем положительные числа. Но умножение абстрактного отрицательного числа на положительное дает только знакомый всем результат. Ведь умножение «больше» на «меньше» дает «меньше».

Правда, в младшем школьном возрасте дети не особо стараются вникать во все математические нюансы.

Хотя, если смотреть правде в глаза, для многих людей даже с высшим образованием многие правила остаются загадкой. Все принимают как должное то, чему их учат учителя, не колеблясь вникают во все трудности, с которыми сопряжена математика. «Меньше» за «меньше» дает «больше» — это знают все без исключения. Это верно как для целых, так и для дробных чисел.

Поделиться:

  • Предыдущая записьКукушкин лен: строение и размножение
  • Следующая записьВал — это что такое? Значение, происхождение, синонимы

×

Рекомендуем посмотреть

Adblock
detector

Отрицательные числа

Отрицательные числа — это числа со знаком минус (−), например −1, −2, −3. Читается как: минус один, минус два, минус три.

Примером применения отрицательных чисел является термометр, показывающий температуру тела, воздуха, почвы или воды. В зимнее время, когда на улице очень холодно, температура бывает отрицательной (или как говорят в народе «минусовой»).

Например, −10 градусов холода:

Обычные же числа, которые мы рассматривали ранее такие как 1, 2, 3 называют положительными. Положительные числа — это числа со знаком плюс (+).

При записи положительных чисел знак + не записывают, поэтому мы и видим привычные для нас числа 1, 2, 3. Но следует иметь ввиду, что эти положительные числа выглядят так: +1, +2, +3.

Координатная прямая

Координатная прямая это прямая линия, на которой располагаются все числа: и отрицательные и положительные. Выглядит следующим образом:

Здесь показаны только числа от −5 до 5. На самом деле координатная прямая бесконечна. На рисунке представлен лишь её небольшой фрагмент.

Числа на координатной прямой отмечают в виде точек. На рисунке жирная чёрная точка является началом отсчёта. Начало отсчёта начинается с нуля. Слева от начала отсчёта отмечают отрицательные числа, а справа — положительные.

Координатная прямая продолжается бесконечно по обе стороны. Бесконечность в математике обозначается символом ∞. Отрицательное направление будет обозначаться символом −∞, а положительное символом +∞. Тогда можно сказать, что на координатной прямой располагаются все числа от минус бесконечности до плюс бесконечности:

(−∞; +∞)

Каждая точка на координатной прямой имеет своё имя и координату. Имя — это любая латинская буква. Координата — это число, которое показывает положение точки на этой прямой. Проще говоря, координата это то самое число, которое мы хотим отметить на координатной прямой.

Например, точка А(2) читается как «точка А с координатой 2« и будет обозначаться на координатной прямой следующим образом:

Здесь A — это имя точки, 2 — координата точки A.


Пример 2. Точка B(4) читается как «точка B с координатой 4« и будет обозначаться на координатной прямой так:

Здесь B — это имя точки, 4 — координата точки B.


Пример 3. Точка M(−3) читается как «точка M с координатой минус три» и будет обозначаться на координатной прямой так:

Здесь M — это имя точки, −3 — координата точки M.

Точки можно обозначать любыми буквами. Но общепринято обозначать их большими латинскими буквами. Более того, начало отчёта, которое по другому называют началом координат принято обозначать большой латинской буквой O

 

Легко заметить, что отрицательные числа лежат левее относительно начала отсчёта, а положительные числа правее.

Существуют такие словосочетания как «чем левее, тем меньше» и «чем правее, тем больше». Наверное, вы уже догадались о чём идёт речь. При каждом шаге влево число будет уменьшаться в меньшую сторону. И при каждом шаге вправо число будет увеличиваться. Стрелка, направленная вправо, указывает на положительное направление отсчёта.


Сравнение отрицательных и положительных чисел

Правило 1. Любое отрицательное число меньше любого положительного числа.

Например, сравним два числа: −5 и 3. Минус пять меньше, чем три, несмотря на то, что пятёрка бросается в глаза в первую очередь, как цифра большая, чем три.

Связано это с тем, что −5 является отрицательным числом, а 3 — положительным. На координатной прямой можно увидеть, где располагаются числа −5 и 3

Видно, что −5 лежит левее, а 3 правее. А мы говорили, что «чем левее, тем меньше». И правило говорит, что любое отрицательное число меньше любого положительного числа. Отсюда следует, что

−5 < 3

«Минус пять меньше, чем три»


Правило 2. Из двух отрицательных чисел меньше то, которое располагается левее на координатной прямой.

Например, сравним числа −4 и −1. Минус четыре меньше, чем минус единица.

Связано это опять же с тем, что на координатной прямой −4 располагается левее, чем −1

Видно, что −4 лежит левее, а −1 правее. А мы говорили, что «чем левее, тем меньше». И правило говорит, что из двух отрицательных чисел меньше то, которое располагается левее на координатной прямой. Отсюда следует, что

−4 < −1

Минус четыре меньше, чем минус единица


Правило 3. Ноль больше любого отрицательного числа.

Например, сравним 0 и −3. Ноль больше, чем минус три. Связано это с тем, что на координатной прямой 0 располагается правее, чем −3

Видно, что 0 лежит правее, а −3 левее. А мы говорили, что «чем правее, тем больше». И правило говорит, что ноль больше любого отрицательного числа. Отсюда следует, что

0 > −3

Ноль больше, чем минус три


Правило 4. Ноль меньше любого положительного числа.

Например, сравним 0 и 4. Ноль меньше, чем 4. Это в принципе ясно и так. Но мы попробуем увидеть это воочию, опять же на координатной прямой:

Видно, что на координатной прямой 0 располагается левее, а 4 правее. А мы говорили, что «чем левее, тем меньше». И правило говорит, что ноль меньше любого положительного числа. Отсюда следует, что

0 < 4

Ноль меньше, чем четыре

Задания для самостоятельного решения

Задание 1. Сравните числа −2 и 1

−2 < 1

Показать решение

Задание 2. Сравните числа −5 и −2

−5 < −2

Показать решение

Задание 3. Сравните числа −5 и −16

−5 > −16

Показать решение

Задание 4. Сравните числа 15 и 20

15 < 20

Показать решение

Задание 5. Сравните числа −7 и 0

−7 < 0

Показать решение

Задание 6. Сравните числа 5 и 0

Показать решение

Задание 7. Сравните числа 5 и 7

Показать решение


Понравился урок?
Вступай в нашу новую группу Вконтакте и начни получать уведомления о новых уроках

Возникло желание поддержать проект?
Используй кнопку ниже

Опубликовано

Умножение целых чисел. — tutomath.ru репетитор по математике

Произведение вы уже проходили в теме умножения натуральных чисел. Отличия произведения натуральных от целых чисел в том, что появляются целые отрицательные числа. Сейчас этой теме мы рассмотрим тему умножение целых чисел подробнее.

Основные понятия, обозначение и смысл произведения целых чисел.

Вспомним, что такое умножение или произведение.
Числа, которые мы умножаем называются множителями, а результат умножения называется произведением.

Обозначается умножение символом таким:
a∙b=c  или a*b=c или a×b=c

Произведение в буквенном написании обозначается как a∙b или c.

Так же вспомним смысл произведения.
Произведение 2∙11=22 можно записать в виде суммы мы сложим 11 раз число 2, это будет выглядеть так:

Правило произведения целых чисел.

Определение:
Произведением двух целых чисел не равных нулю называют произведение их модулей и результат будет со знаком плюс, если эти числа одинаковых знаков, и со знаком минус, если они разных знаков.

Самое главное в произведении целых чисел это правильно посчитать знак ответа. Например, оба множителя могут быть положительными или оба отрицательными числами, или один множитель положительный, а другой отрицательный.

Нужно запомнить:

Плюс на плюс дает плюс.
“+ ∙ + = +”

Минус на минус дает плюс.
“– ∙ – =+”

Минус на плюс дает минус.
“– ∙ + = –”

Плюс на минус дает минус.
“+ ∙ – = –”

Каждый случай ниже разберем подробно.

Умножение или произведение положительных целых чисел.

В данном случае мы умножаем два числа положительных знаков, поэтому тут все просто “ плюс на плюс дает плюс”. Произведение положительных целых чисел дает в результате положительное целое число. Рассмотрим пример:

Для наглядности разберем умножение со знаками.
(+5)∙(+8)=(+40)
В умножении не принято писать знак “+”, поэтому его можно опустить. Если перед число не стоит ни какого знака, то считается то перед этим числом стоит знак “+”.
5∙8=40

Умножение отрицательных целых чисел.

Правило умножения двух целых отрицательных чисел:
При умножении двух отрицательных целых чисел, будет равно произведению модулей этих чисел.

|-a|=a и |-b|=b
-a∙(-b)=a∙b

Или другими словами “минус на минус дает плюс”. При произведении двух отрицательных чисел, ответ будет равен положительному целому числу.

Пример:
Вычислите произведение целых чисел -12∙(-3).

Решение:
Два минуса при умножении дают в результате плюс. В ответе число будет с плюсом.
-12∙(-3)=36

Ответ: 36

Произведение целых чисел с разными знаками.

Не важен порядок множителей положительное число умножаем на отрицательное или отрицательное число умножаем на положительное, в результате всегда будет отрицательное целое число.

Правило умножения двух целых чисел с разными знаками:
При умножении двух целых чисел с разными знаками, их произведение будет равно целому отрицательному числу.

Если упростить определение то, обычно говорят:
“Минус на плюс дает минус”.
“Плюс на минус дает минус”.

Разберем пример:
Вычислить произведение целых чисел.
-4∙6=-24

А теперь докажем правильность этого решения.
-4+(-4)+(-4)+(-4)+(-4)+(-4)=-4∙6=-24
Шесть раз сложили число (-4).

Такой же ответ будет, если поменять местами числа.
6∙(-4)=-24

Пример:
-34∙2=-68

Умножение целого числа на нуль.

Правило умножения целых чисел на нуль.
Если любое целое число умножить на нуль, ответ будет равен нулю.
a∙0=0  или 0∙a=0

Пример:
Найдите произведение целого положительного числа 209 на нуль.

Решение:
209∙0=0

Пример:
Найдите произведение целого отрицательного числа (-39) на нуль.

Решение:
0∙(-39)=0

Умножение целого числа на 1.

Правило умножения целого числа на единицу:
Произведение целого числа a и 1 равно a.
a∙1=a или 1∙a=a

Пример:
Вычислить произведение положительного целого числа 49 и единицы.

Решение:
49∙1=49

Пример:
Вычислить произведение отрицательного целого числа (-35 860) и единицы.

Решение:
1∙(-35 860)=-35 860

Пример:
Найдите произведение нуля и единицы.

Решение:
0∙1=0

Проверка результата умножения целых чисел.

Не всегда мы выполняем умножение простых чисел, бывают число объемные и сложные, поэтому нужно уметь проверять правильность выполненного умножения.
Как проверить результат умножения?

Умножение проверяется делением. Мы делим произведение на один из множителей.

Например:
Выполните умножение и сделайте проверку.
5∙12=60

5 – множитель;
12 – множитель;
60 – произведение.

Проверка:
60:12=5  или  60:5=12

Умножение или произведение нескольких целых чисел.

Чтобы посчитать произведение нескольких целых чисел, нужно умножать числа по парно или последовательно, например:
(-3) ∙5∙(-11) ∙(-9) ∙1=((-3) ∙5)∙((-11) ∙(-9)) ∙1=((-15) ∙99) ∙1=(-1485) ∙1=-1485

Сначала сгруппировали по два числа ((-3) ∙5) и ((-11) ∙(-9)), потом ((-15) ∙99) и нашли ответ.

При перемножении целых чисел, результат всегда будет целым числом.

Вопросы по теме:
Как влияет при умножении на целое число (-1)?
Ответ: так как (-1) отрицательное число, при умножении на целое число происходит смена знака числа.
Пример: (-1) ∙3=-3 . Число 3 было со знаком “+”, а стало со знаком “–”.
Еще пример: (-1) ∙(-5)=5 . Число (-5) было со знаком “–”, а стало со знаком “+”.

Пример №1:
Найти произведение двух целых чисел: а) (-2) ∙235 б) (-34) ∙(-17) в) 1∙(-12) г) 0∙4983

Решение:
а) (-2) ∙235=-470
б) (-34) ∙(-17)=578
в) 1∙(-12)=-12
г) 0∙4983=0

Пример №2:
Чему равно произведение последовательных целых чисел, начинающихся числом -100 и оканчивающихся числом 100?

Решение:
Между числами -100 и 100 находится нуль, а любое число, умноженное на 0 равно 0. Поэтому произведение последовательных целых чисел от -100 до 100 равно 0.
Ответ: 0.

Пример №3:
Чему равно произведение всех целых чисел?

Решение:
Целые числа состоят из целых положительных и отрицательных чисел, а также нуля. При умножении любого числа на нуль будет 0. Поэтому произведение всех целых чисел равно 0.
Ответ: 0.

ИСТОРИЯ ВОЗНИКНОВЕНИЯ ОТРИЦАТЕЛЬНЫХ И ПОЛОЖИТЕЛЬНЫХ ЧИСЕЛ

  • Авторы
  • Руководители
  • Файлы работы
  • Наградные документы

Семенов Д. У. 1


1

Джамбаева Ф.Н. 1


1

Автор работы награжден дипломом победителя III степени

Диплом школьникаСвидетельство руководителя

Текст работы размещён без изображений и формул.
Полная версия работы доступна во вкладке «Файлы работы» в формате PDF

 Введение

Мир чисел очень загадочен и интересен. Числа очень важны в нашем мире. Я хочу узнать как можно больше о происхождении чисел, об их значении в нашей жизни. Как их применять и какую роль они играют в нашей жизни?

В прошлом году на уроках математики мы начали изучать тему «Положительные и отрицательные числа». У меня возник вопрос, когда возникли отрицательные числа, в какой стране, какие ученые занимались этим вопросом. В Википедии я прочитал, что отрицательное число — элемент множества отрицательных чисел, которое (вместе с нулём) появилось в математике при расширении множества натуральных чисел. Цель расширения: обеспечить выполнение операции вычитания для любых чисел. В результате расширения получается множество (кольцо) целых чисел, состоящее из положительных (натуральных) чисел, отрицательных чисел и нуля.

В итоге я решил исследовать историю возникновения отрицательных чисел.

Целью данной работы является исследование истории возникновения отрицательных и положительных чисел.

Объект исследования — отрицательные числа и положительные числа

История положительных и отрицательных чисел

Люди долго не могли привыкнуть к отрицательным числам. Отрицательные числа казались им непонятными, ими не пользовались, просто не видели в них особого смысла. Эти числа появились значительно позже натуральных чисел и обыкновенных дробей.

Первые сведения об отрицательных числах встречаются у китайских математиков во II в. до н. э. и то, были известны лишь правила сложения и вычитания положительных и отрицательных чисел; правила умножения и деления не применялись.

Положительные количества в китайской математике называли «чен», отрицательные – «фу»; их изображали разными цветами: «чен» — красным, «фу» — черным. Это можно заметить в книге «Арифметика в девяти главах» (Автор Чжан Цань). Такой способ изображения использовался в Китае до середины XII столетия, пока Ли Е не предложил более удобное обозначение отрицательных чисел – цифры, которые изображали отрицательные числа, перечеркивали черточкой наискось справа налево.

Лишь в VII в. индийские математики начали широко использовать отрицательные числа, но относились к ним с некоторым недоверием. Бхасхара прямо писал: «Люди не одобряют отвлеченных отрицательных чисел. ..». Вот как индийский математик Брахмагупта излагал правила сложения и вычитания: «имущество и имущество есть имущество, сумма двух долгов есть долг; сумма имущества и нуля есть имущество; сумма двух нулей есть нуль… Долг, который отнимают от нуля, становится имуществом, а имущество – долгом. Если нужно отнять имущество от долга, а долг от имущества, то берут их сумму». «Сумма двух имуществ есть имущество».

(+х) + (+у) = +(х + у)‏ (-х) + (-у) = — (х + у)‏

(-х) + (+у) = — (х — у)‏ (-х) + (+у) = +(у — х)‏

0 – (-х) = +х 0 – (+х) = -х

Индийцы называли положительные числа «дхана» или «сва» (имущество), а отрицательные – «рина» или «кшайя» (долг). Индийские ученые, стараясь найти и в жизни образцы такого вычитания, пришли к толкованию его с точки зрения торговых расчетов. Если купец имеет 5000 р. и закупает товара на 3000 р., у него остается 5000 — 3000 = 2000, р. Если же он имеет 3000 р., а закупает на 5000 р., то он остается в долгу на 2000 р. В соответствии с этим считали, что здесь совершается вычитание 3000 — 5000, результатом же является число 2000 с точкой наверху, означающее «две тысячи долга». Толкование это носило искусственный характер, купец никогда не находил сумму долга вычитанием 3000 — 5000, а всегда выполнял вычитание 5000 — 3000.

Чуть позже в Древней Индии и Китае догадались вместо слов «долг в 10 юаней» писать просто «10 юаней», но рисовать эти иероглифы черной тушью. А знаков «+» и «–» в древности не было ни для чисел, ни для действий.

Греки тоже поначалу знаков не использовали. Древнегреческий ученый Диофант вообще не признавал отрицательные числа, и если при решении уравнения получался отрицательные корень, то он отбрасывал его как «недоступный». И Диофант старался так сформулировать задачи и составлять уравнения, чтобы избежать отрицательных корней, но вскоре Диофант Александрийский стал обозначать вычитание знаком.

Правила действий с положительными и отрицательными числами были предложены уже в III веке в Египте. Введение отрицательных величин впервые произошло у Диофанта. Он даже использовал специальный символ для них. В то же время Диофант употребляет такие обороты речи, как «Прибавим к обеим сторонам отрицательное», и даже формулирует правило знаков: «Отрицательное, умноженное на отрицательное, дает положительное, тогда как отрицательное, умноженное на положительное, дает отрицательное».

В Европе отрицательными числами начали пользоваться с XII–XIII вв., но до XVI в. большинство ученых считали их «ложными», «мнимыми» или «абсурдными», в отличие от положительных чисел – “истинных”. Положительные числа так же толковались как «имущество», а отрицательные – как «долг», «недостача». Даже знаменитый математик Блез Паскаль утверждал, что 0 − 4 = 0, так как ничто не может быть меньше, чем ничто. В Европе к идее отрицательного количества достаточно близко подошел в начале XIII столетия Леонардо Фибоначчи Пизанский.

На состязании в решении задач с придворными математиками Фридриха II Леонардо Пизанскому было предложено решить задачу: требовалось найти капитал нескольких лиц. Фибоначчи получил отрицательное значение. «Этот случай, — сказал Фибоначчи, — невозможен, разве только принять, что один имел не капитал, а долг». Однако в явном виде отрицательные числа применил впервые в конце XV столетия французский математик Шюке. Автор рукописного трактата по арифметике и алгебре «Наука о числах в трёх частях». Символика Шюке приближается к современной.

Признанию отрицательных чисел способствовали работы французского математика, физика и философа Рене Декарта. Он предложил геометрическое истолкование положительных и отрицательных чисел – ввел координатную прямую. (1637 г.).

Положительные числа изображаются на числовой оси точками, лежащими вправо от начала 0, отрицательные – влево. Геометрическое истолкование положительных и отрицательных чисел способствовало к их признанию.

В 1544 году немецкий математик Михаил Штифель впервые рассматривает отрицательные числа как числа, меньшие нуля (т. е. « меньшие, чем ничто »). С этого момента отрицательные числа рассматриваются уже не как долг, а совсем по-новому. Сам Штифель писал: «Нуль находится между истинными и абсурдными числами…»

Почти одновременно со Штифелем защищал идею отрицательных чисел Бомбелли Раффаэле (около 1530—1572), итальянский математик и инженер, переоткрывший сочинение Диофанта.

Так же и Жирар считал отрицательные числа вполне допустимыми и полезными, в частности, для обозначения недостачи чего-либо.

Всякий физик постоянно имеет дело с числами: он всегда что-то измеряет, вычисляет, рассчитывает. Везде в его бумагах — числа, числа и числа. Если приглядеться к записям физика, то обнаружится, что при записи чисел он часто использует знаки «+» и «-«. (Например: термометр, шкала глубин и высот)

Только в начале XIX в. теория отрицательных чисел закончила свое развитие, и «абсурдные числа» получили всеобщее признание.

Определение понятия числа

В современном мире человек постоянно пользуется числами, даже не задумываясь об их происхождении. Без знания прошлого нельзя понять настоящее. Число является одним из основных понятий математики. Понятие числа развивалось в тесной связи с изучением величин; эта связь сохраняется и теперь. Во всех разделах современной математики приходится рассматривать разные величины и пользоваться числами. Число — абстракция, используемая для количественной характеристики объектов. Возникнув ещё в первобытном обществе из потребностей счёта, понятие числа изменялось и обогащалось и превратилось в важнейшее математическое понятие.

Существует большое количество определений понятию «число».

Первое научное определение числа дал Евклид в своих «Началах», которое он, очевидно, унаследовал от своего соотечественника Эвдокса Книдского (около 408 – около 355 гг. до н. э.): «Единица есть то, в соответствии с чем каждая из существующих вещей называется одной. Число есть множество, сложенное из единиц». Так определял понятие числа и русский математик Магницкий в своей «Арифметике» (1703 г.). Еще раньше Евклида Аристотель дал такое определение: «Число есть множество, которое измеряется с помощью единиц». В своей «Общей арифметике» (1707 г) великий английский физик, механик, астроном и математик Исаак Ньютон пишет: «Под числом мы подразумеваем не столько множество единиц, сколько абстрактное отношение какой-нибудь величины к другой величине такого же рода, взятой за единицу. Число бывает трех видов: целое, дробное и иррациональное. Целое число есть то, что измеряется единицей; дробное – кратной частью единицы, иррациональное – число, не соизмеримое с единицей».

Мариупольский математик С.Ф.Клюйков также внес свой вклад в определение понятия числа: «Числа – это математические модели реального мира, придуманные человеком для его познания». Он же внес в традиционную классификацию чисел так называемые «функциональные числа», имея в виду то, что во всем мире обычно именуют функциями.

Натуральные числа возникли при счете предметов. Об этом я узнала в 5 классе. Затем я узнала, что потребность человека измерять величины не всегда выражается целым числом. После расширения множества натуральных чисел до дробных стало возможным делить любое целое число на другое целое число (за исключением деления на нуль). Появились дробные числа. Вычитать же целое число из другого целого числа, когда вычитаемое больше уменьшаемого, долгое время казалось невозможным. Интересным для меня оказался тот факт, что долгое время многие математики не признавали отрицательных чисел, считая, что им не соответствуют какие-либо реальные явления.

Происхождение слов «плюс» и «минус»

Термины произошли от слов plus – «больше», minus – «меньше». Сначала действия обозначали первыми буквами p; m. Многие математики предпочитали или Возникновение современных знаков «+», «–» не совсем ясно. Знак «+», возможно, происходит от сокращенной записи et, т.е. «и». Впрочем, может быть он возник из торговой практики: проданные меры вина отмечались на бочке «–», а при восстановлении запаса их перечеркивали, получался знак «+».

Италии ростовщики, давая деньги в долг, ставили перед именем должника сумму долга и черточку, вроде нашего минуса, а когда должник возвращал деньги, зачеркивали ее, получалось что-то вроде нашего плюса.

Современные знаки «+» и появились в Германии в последнее десятилетие XVв. в книге Видмана, которая была руководством по счету для купцов (1489г. ). Чех Ян Видман уже писал «+» и «–» для сложения и вычитания.

Чуть позднее немецкий ученый Михель Штифель написал «Полную Арифметику», которая была напечатана в 1544 году. В ней встречаются такие записи для чисел: 0-2; 0+2; 0-5; 0+7. Числа первого вида он назвал «меньше, чем ничего» или «ниже, чем ничего». Числа второго вида назвал «больше, чем ничего» или «выше, чем ничего». Вам, конечно, понятны эти названия, потому что «ничего» – это 0.

Отрицательные числа в Египте

Однако, не смотря на такие сомнения, правила действий с положительными и отрицательными числами были предложены уже в III веке в Египте. Введение отрицательных величин впервые произошло у Диофанта. Он даже использовал специальный символ для них (сейчас мы в этом качестве используем знак «минус»). Правда, ученые спорят, обозначал ли символ Диофанта именно отрицательное число или просто операцию вычитания, потому что у Диофанта отрицательные числа не встречаются изолированно, а только в виде разностей положительных; и в качестве ответов в задачах он рассматривает только рациональные положительные числа. Но в то же время Диофант употребляет такие обороты речи, как «Прибавим к обеим сторонам отрицательное», и даже формулирует правило знаков: «Отрицательное, умноженное на отрицательное, дает положительное, тогда как отрицательное, умноженное на положительное, дает отрицательное» (то, что сейчас обычно формулируют: «Минус на минус дает плюс, минус на плюс дает минус»).

(–) (–) = (+), (–) (+) = (–).

Отрицательные числа в Древней Азии

Положительные количества в китайской математике называли «чен», отрицательные – «фу»; их изображали разными цветами: «чен» — красным, «фу» — черным. Такой способ изображения использовался в Китае до середины XII столетия, пока Ли Е не предложил более удобное обозначение отрицательных чисел – цифры, которые изображали отрицательные числа, перечеркивали черточкой наискось справа налево. Индийские ученые, стараясь найти и в жизни образцы такого вычитания, пришли к толкованию его с точки зрения торговых расчетов.

Если купец имеет 5000 р. и закупает товара на 3000 р., у него остается 5000 — 3000 = 2000, р. Если же он имеет 3000 р., а закупает на 5000 р., то он остается в долгу на 2000 р. В соответствии с этим считали, что здесь совершается вычитание 3000 — 5000, результатом же является число 2000 с точкой наверху, означающее «две тысячи долга».

Толкование это носило искусственный характер, купец никогда не находил сумму долга вычитанием 3000 — 5000, а всегда выполнял вычитание 5000 — 3000. Кроме того, на этой основе можно было с натяжкой объяснить лишь правила сложения и вычитания «чисел с точками», но никак нельзя было объяснить правила умножения или деления.

В V-VI столетиях отрицательные числа появляются и очень широко распространяются в индийской математике. В Индии отрицательные числа систематически использовали в основном так, как это мы делаем сейчас. Индийские математики используют отрицательные числа с VII в. н. э.: Брахмагупта сформулировал правила арифметических действий с ними. В его произведении мы читаем: « имущество и имущество есть имущество, сумма двух долгов есть долг; сумма имущества и нуля есть имущество; сумма двух нулей есть нуль… Долг, который отнимают от нуля, становится имуществом, а имущество – долгом. Если нужно отнять имущество от долга, а долг от имущества, то берут их сумму».

Индийцы называли положительные числа «дхана» или «сва» (имущество), а отрицательные – «рина» или «кшайя» (долг). Впрочем, и в Индии с пониманием и принятием отрицательных чисел были проблемы.

Отрицательные числа в Европе

Не одобряли их долго и европейские математики, потому что истолкование «имущество-долг» вызывало недоумения и сомнения. В самом деле, как можно «складывать» или «вычитать» имущества и долги, какой реальный смысл может иметь «умножение» или «деление» имущества на долг? (Г. И. Глейзер, История математики в школе IV-VI классы. Москва, Просвещение, 1981)

Вот почему с большим трудом завоевали себе место в математике отрицательные числа. В Европе к идее отрицательного количества достаточно близко подошел в начале XIII столетия Леонардо Фибоначчи Пизанский, однако в явном виде отрицательные числа применил впервые в конце XV столетия французский математик Шюке. Автор рукописного трактата по арифметике и алгебре «Наука о числах в трёх частях». Символика Шюке приближается к современной (Математический энциклопедический словарь. М., Сов. энциклопедия, 1988)

Современное истолкование отрицательных чисел

В 1544 году немецкий математик Михаил Штифель впервые рассматривает отрицательные числа как числа, меньшие нуля (т. е. « меньшие, чем ничто »). С этого момента отрицательные числа рассматриваются уже не как долг, а совсем по-новому. Сам Штифель писал: «Нуль находится между истинными и абсурдными числами…» (Г.И. Глейзер, История математики в школе IV-VI классы. Москва, Просвещение, 1981)

После этого Штифель полностью посвящает свою работу математике, в которой он был гениальным самоучкой. Один из первых в Европе после Николы Шюке начал оперировать отрицательными числами.

Знаменитый французский математик Рене Декарт в «Геометрии» (1637 год) описывает геометрическое истолкование положительных и отрицательных чисел; положительные числа изображаются на числовой оси точками, лежащими вправо от начала 0, отрицательные – влево. Геометрическое истолкование положительных и отрицательных чисел привело к более ясному пониманию природы отрицательных чисел, способствовало их признанию.

Почти одновременно со Штифелем защищал идею отрицательных чисел Р. Бомбелли Раффаэле (около 1530—1572), итальянский математик и инженер, переоткрывший сочинение Диофанта.

Бомбелли и Жирар, напротив, считали отрицательные числа вполне допустимыми и полезными, в частности, для обозначения недостачи чего-либо. Современное обозначение положительных и отрицательных чисел со знаками « + » и « — » применил немецкий математик Видман. Выражение «ниже, чем ничего» показывает, что Штифель и некоторые другие мысленно воображали положительные и отрицательные числа точками на вертикальной шкале (вроде шкалы термометра). Развитое затем математиком А. Жираром представление об отрицательных числах как о точках на некоторой прямой, располагающихся по другую сторону от нуля, чем положительные, оказалось решающим в обеспечении этим числам прав гражданства, особенно в результате развития метода координат у П. Ферма и Р. Декарта.

Вывод

В своем работе я исследовал историю возникновения отрицательных чисел. В ходе исследования я сделал вывод:

Современная наука встречается с величинами такой сложной природы, что для их изучения приходится изобретать все новые виды чисел.

При введении новых чисел большое значение имеют два обстоятельства:

а) правила действий над ними должны быть полностью определены и не вели к противоречиям;

б) новые системы чисел должны способствовать или решению новых задач, или усовершенствовать уже известные решения.

К настоящем у времени существует семь общепринятых уровней обобщения чисел: натуральные, рациональные, действительные, комплексные, векторные, матричные и трансфинитные числа. Отдельными учеными предлагается считать функции функциональными числами и расширить степень обобщения чисел до двенадцати уровней.

Все эти множества чисел я постараюсь изучить.

Приложение

СТИХОТВОРЕНИЕ

«Сложение отрицательных чисел и чисел с разными знаками»

Если уж захочется вам сложить

Числа отрицательные, нечего тужить:

Надо сумму модулей быстренько узнать,

К ней потом знак «минус» взять да приписать.

Если числа с разными знаками дадут,

Чтоб найти их сумму, все мы тут как тут.

Больший модуль быстро очень выбираем.

Из него мы меньший вычитаем.

Самое же главное – знак не позабыть!

— Вы какой поставите? – мы хотим спросить

— Вам секрет откроем, проще дела нет,

Знак, где модуль больше, запиши в ответ.

Правила сложения положительных и отрицательных чисел

Минус с минусом сложить,

Можно минус получить.

Если сложишь минус, плюс,

То получится конфуз?!

Знак числа ты выбирай

Что сильнее, не зевай!

Модули их отними,

Да все числа помири!

— Правила умножения можно истолковать и таким образом:

«Друг моего друга — мой друг»: + ∙ + = + .

«Враг моего врага — мой друг»: ─ ∙ ─ = +.

«Друг моего врага — мой враг»: + ∙ ─ = ─.

«Враг моего друга – мой враг»: ─ ∙ + = ─.

Знак умножения есть точка, в ней три знака:

Прикрой из них два, третий даст ответ.

Например.

Как определить знак произведения 2∙(-3)?

Закроем руками знаки «плюс» и «минус». Остаётся знак «минус»

Список литературы

  1. «История древнего мира», 5 класс. Колпаков, Селунская.

  2. «История математики в древности», Э. Кольман.

  3. «Справочник школьника». ИД «ВЕСЬ», Санкт-Петербург. 2003 г.

  4. Большая математическая энциклопедия. Якушева Г.М. и др.

  5. Большая математическая энциклопедия. Якушева Г.М. и др.

  6. Вигасин А.А,.Годер Г.И., «История древнего мира» учебник 5 класса, 2001г.

  7. Википедия. Свободная энциклопедия.

  8. Возникновение и развитие математической науки: Кн. Для учителя. – М.: Просвещение, 1987.

  9. Возникновение и развитие математической науки: Кн. Для учителя. – М.: Просвещение, 1987.

  10. Гельфман Э. Г. «Положительные и отрицательные числа», учебное пособие по математике для 6-го класса, 2001.

  11. Глав. ред. М. Д. Аксёнова. – М.: Аванта+,1998.

  12. Глав. ред. М. Д. Аксёнова. – М.: Аванта+,1998.

  13. Глейзер Г. И. «История математики в школе», Москва, «Просвещение», 1981 г.

  14. Детская энциклопедия «Я познаю мир», Москва, «Просвещение», 1995г.

  15. История математики в школе , IV-VI классы. Г.И. Глейзер, Москва, Просвещение, 1981.

  16. История математики в школе , IV-VI классы. Г.И. Глейзер, Москва, Просвещение, 1981.

  17. М.: Филол. О-во «СЛОВО»: ОЛМА-ПРЕСС, 2005.

  18. Малыгин К.А.

  19. Математический энциклопедический словарь. М., Сов. энциклопедия, 1988.

  20. Нурк Э.Р., Тельгмаа А.Э. «Математика 6 класс», Москва, «Просвещение»,1989г

  21. Учебник 5 класс. Виленкин, Жохов, Чесноков, Шварцбурд.

  22. Фридман Л. М.. «Изучаем математику», учебное издание, 1994 г.

  23. Э.Г. Гельфман и др., Положительные и отрицательные числа в театре Буратино. Учебное пособие по математике для 6 класса. 3-е издание, испр., — Томск: Издательство Томского университета, 1998г.

  24. Энциклопедия для детей. Т.11. Математика

17

Просмотров работы: 24855

Действия с нулём

В математике число ноль занимает особое место. Дело в том, что оно, по сути дела, означает «ничто», «пустоту», однако его значение действительно трудно переоценить. Для этого достаточно вспомнить хотя бы то, что именно с нулевой отметки начинается отсчет координат положения точки в любой системе координат.

 

Ноль широко используется в десятичных дробях для определения значений «пустых» разрядов, находящихся как до, так и после запятой. Кроме того, именно с ним связано одно из основополагающих правил арифметики, гласящее о том, что на ноль делить нельзя. Его логика, собственно говоря, проистекает из самой сути этого числа: действительно, невозможно представить, чтобы некая отличное от него значение (да и само оно – тоже) было разделено на «ничто».

Примеры вычисления

С нулем осуществляются все арифметические действия, причем в качестве его «партнеров» по ним могут использоваться целые числа, обычные и десятичные дроби, причем все они могут иметь как положительное, так и отрицательное значение. Приведем примеры их осуществления и некоторые пояснения к ним.

Сложение

При прибавлении нуля к некоторому числу (как целому, так и к дробному, как к положительному, так и к отрицательному) его значение остается абсолютно неизменным.

Пример 1

Двадцать четыре плюс ноль равняется двадцать четыре.

24 + 0 = 24

Пример 2

Семнадцать целых три восьмых плюс ноль равняется семнадцать целых три восьмых.

17

3

8

+ 0 = 17

3

8

Вычитание

При вычитании нуля из некоторого числа (целого, дробного, положительного или отрицательного) оставляет его полностью неизменным.

Пример 1

Две тысячи сто пятьдесят два минус ноль равняется две тысячи сто пятьдесят два.

21520 = 2152

Пример 2

Сорок одна целая три пятых минус ноль равняется сорок одна целая три пятых.

41

3

5

– 0 = 41

3

5

Умножение

При умножении любого числа (целого, дробного, положительного или отрицательного) на ноль получается ноль.

Пример 1

Пятьсот восемьдесят шесть умножить на ноль равняется ноль.

586 × 0 = 0

Пример 2

Ноль умножить на сто тридцать пять целых шесть седьмых равняется ноль.

0 × 135 = 0

Пример 3

Ноль умножить на ноль равняется ноль.

0 × 0 = 0

Деление

Правила деления чисел друг на друга в тех случаях, когда одно из них представляет собой ноль, различаются в зависимости от того, в какой именно роли выступает сам ноль: делимого или делителя?

В тех случаях, когда ноль представляет собой делимое, результат всегда равен ему же, причем вне зависимости от значения делителя.

Пример 1

Ноль разделить на двести шестьдесят пять равняется ноль.

0 : 265 = 0

Пример 2

Ноль разделить на семнадцать пятьсот девяносто шестых равняется ноль.

0 :

17

596

= 0

Делить ноль на ноль согласно правилам математики нельзя. Это означает, что при совершении такой процедуры частное является неопределенным. Таким образом, теоретически оно может представлять собой абсолютно любое число.

0 : 0 = 8 ибо 8 × 0 = 0

В математике такая задача, как деление нуля на ноль, не имеет никакого смысла, поскольку ее результат представляет собой бесконечное множество. Это утверждение, однако, справедливо в том случае, если не указаны никакие дополнительные данные, которые могут повлиять на итоговый результат.

Таковые, при их наличии, должны состоять в том, чтобы указывать на степень изменения величины как делимого, так и делителя, причем еще до наступления того момента, когда они превратились в ноль. Если это определено, то такому выражению, как ноль разделить на ноль, в подавляющем большинстве случаев можно придать некий смысл.

Репетитор по математике о работе с правилом вычитания отрицательных чисел — Колпаков Александр Николаевич

Выработка вычислительных навыков – важнейшая цель, преследуемая программами по математике с 1 по 6 класс. От того, насколько быстро и правильно ребенок научится выполнять арифметические действия, будет зависеть скорость выполнения им логических (смысловых) операции в старших класах и уровень понимания предмета в целом. Репетитор по математике довольно часто сталкивается с вычислительными проблемами учащихся, мешающими добиваться высоких результатов.

С какими только учениками не приходится работать репетитору. Родителям нужна подготовка к ЕГЭ по математике, а их чадо не может разобраться в обыкновенных дробях или путается в отрицательных числах. Какие действия должны предприниматься репетитором по математике в таких случаях? Как помочь ученику? Времени на неспешное и последовательное изучение правил у репетитора нет, поэтому традиционные методы часто приходится заменять некими искусственными «полуфабрикатами-ускорителями», если можно так выразиться. В этой статье я опишу один из возможных путей формирования навыка выполнения действий с отрицательными числами, а именно вычитания таковых.

Предположим, что репетитор по математике имеет удовольствие работать с очень слабым учеником, знания которого дальше простейших вычислений с положительными числами не распространяются. Предположим также, что репетитору удалось объяснить законы сложения и вплотную подойти к правилу a-b=a+(-b). Какие моменты должен учесть репетитор по математике?

Сведения вычитания к сложению не является простым и очевидным преобразованием. Учебники предлагают строгие и точные математические формулировки: «Чтобы из числа «а» вычесть число «b» надо к числу «а» прибавить число, противоположное к « b». Формально к тексту не придерешься, но как только он начинает применяться репетитором по математике в качестве инструкции к выполнению конкретных вычислений — возникают проблемы. Одна только фраза чего стоит: «Чтобы вычесть – надо прибавить». Без внятного комментария репетитора ученик не разберется. В самом деле, что же делать: вычитать или складывать?

Если работать с правилом согласно замыслу авторов учебника, то помимо отработки понятия «противоположное число», нужно научить школьника соотносить обозначения «а» и «b» с реальными числами в примере. А на это потребуется время. Учитывая еще и тот факт, что ученик думает и пишет одновременно, задача репетитора по математике еще большет усложняется. Хорошей зрительной, смысловой и двигательной памятью слабый ученик не обладает, а поэтому лучше предложить альтернативный текст правила:

Чтобы из первого числа вычесть второе, нужно
А) Первое число переписать
Б) Поставить плюс
B) Заменить знак второго числа на противоположный
Г) Сложить полученные числа

Здесь этапы алгоритма четко разделяются по пунктам и не привязываются к буквенным обозначениям.

По ходу решения практического задания на переводы, репетитор по математике перечитывает этот текст ученику по нескольку раз (для запоминания). Я советую записать его в теоретическую тетрадь. Только после отработки правила перехода к сложению можно записать общую форму a-b=a+(-b)

Движение знаков «минус» и «плюс» в голове ребенка (как маленького, так и слабого взрослого) в чем-то напоминает броуновское. Навести порядок в этом хаосе репетитору по математике нужно как можно быстрее. В процессе решения примеров применяются опорные подсказки (словесные и визуальные), которые в сочетании аккуратным и подробным офофрмлением делают свое дело. Нужно помнить, что каждое слово, произнесенное репетитором по математике в момент решения любой задачи несет или подсказку или помеху. Каждая фраза анализируется ребенком на предмет установления связи с теми или иным математическим объектом (явлением) и его образом на бумаге.

Типичная проблема слабых школьников — отделение знака действия от знака числа в нем участвующего. Одинаковый визуальный образ мешает распознавать уменьшаемое «a» и вычитаемое «b» в разности a-b. Когда в процессе объяснений репетитор по математике читает выражение, нужно следить за тем, чтобы вместо «-» употреблялось слово «вычесть». Это обязательно! Например, запись следует читать так: «Из минус пяти вычесть минус три». Нельзя забывать и о правиле перевода в сложение: «Чтобы из числа «а» вычесть число «b» надо … ».

Если у репетитора по математике постоянно слетит с языка «минус 5 минус минус 3», то понятно, что ученику будет труднее представить себе структуру примера. Однозначное соответствие между словом и арифметическим действием помогает репетитору по математике точно транслировать информацию.

Как репетитору объяснить переход к сложению?

Конечно, можно обратиться к определению понятия «вычесть» и искать число, которое надо прибавить к «b» для получения «а». Однако, слабый ученик мыслит далек от строгой математики и репетитору в работе с ним потребуются некие аналогии с простыми действиями. Я часто говорю своим шестиклашкам: «В математике нет такого арифметического действия, как «разность». Запись 5 – 3 является простым обозначением результата сложения 5+(-3). Знак «плюс» просто опускают и не пишут».

Дети удивляются словам репетитора и непроизвольно запоминают, что нельзя вычитать числа напрямую. Репетитор по математике объявляет 5 и -3 слагаемыми, и для большей убудительности своих слов сравнивает результаты действий 5-3 и 5+(-3). После этого записывается тождество a-b=a+(-b)

Каков бы ни был ученик, и сколько бы времени не отводилось репетитору по математике на занятия с ним, нужно вовремя отработать понятие «противоположное число». Отдельного внимания репетитора по математике заслуживает запись «-х». Ученик 6 класса должен усвоить, что она отображает не отрицательное число, а противоположное к иксу.

Необходимо отдельно остановиться на вычислениях с двумя знаками «минус», расположенными рядом. Возникает проблема понимания операции их одновременного удаления. Нужно аккуратно пройти по всем пунктам изложенного алгоритма перехода к сложению. Будет лучше, если в работе с разностью -5- (-3) до каких-либо комментариев репетитор по математике выделит числа -5 и -3 в рамочку или подчеркнет их. Это поможет ученику выделить компоненты действия.

Нацеленность репетитора по математике на запоминание


Надежное запоминание – результат практического применения математических правил, поэтому репетитору важно обеспечить хорошую плотность самостоятельно решенных примеров. Для улучшения устойчиваости запоминания можно призвать на помощь визуальные подсказки — фишечки. Например, интересный способ перевовода вычитания отрицательного числа в сложение. Репетитор по математике соединяет два минуса одной линией (как показано на рисунке), и взору ученика открывается знак «плюс» (в пересечении со скобкой).

Для предотвращения рассеивания внимания я рекомендую репетиторам по математике выделять уменьшаемое и вычитаемое рамками. Если репетитор по математике использует рамки или кружочки для выделения компонентов арифметического действия, то ученик легче и быстрее найчится видеть структуру примера и соотносить ее с соответствующим правилом. Не следует располагать кусочки целого объекта при оформлении решений на разных строчках тетрадного листа, а также приступать к сложению до тех пор, пока оно не будет записано. Все действия и переходы в обязательном порядке показываются (по крайней мере на старте изучения темы).

Некоторые репетиторы по математике стремятся к 100% точному обоснованию правил перевода, считая эту стратегию единственно правильной и полезной для формирования вычислительных навыков. Однако, практика показывает, что этот путь не всегда приносит хорошие дивиденды. Потребность в осознании того, что человек делает, чаще всего появляется после запоминания этапов применяемого алгоритма и практического закрепления вычислительных операций.

Крайне важно отработать переход к сумме в длинном числовом выражении с несколькими вычитаниями, например . Перед тем, как приступить к подсчету или преобразованию, я заставляю ученика обвести в кружочки числа вместе с их знаками, расположенными слева. На рисунке показан пример того, как репетитор по математкие выделяет слагаемые Для очень слабых шестиклассников можно дополнительно подкрашивать кружочки. Для положительных слагаемых использовать один цвет, а для отрицательных другой. В особых случаях беру в руки ножницы и режу выражение на кусочки. Их можно произвольно перекладывать, иммитируя таким образом перестановку слагаемых. Ребенок увидит, что знаки перемещаются вместе с самими слагаемыми. То есть, если знак минус стоял слева от числа 5, то куда бы мы не перекладывали соответствующую карточку, он от пятерки не оторвется.

Колпаков А.Н. Репетитор по математике 5-6 класс. Москва. Строгино.

Почему отрицательное время является отрицательным положительным? – The Reflective Educator

Возможны разные ответы на этот вопрос, в зависимости от необходимого стандарта доказательства и исходных знаний, которые человек привносит в вопрос.

 

Математическая согласованность и закономерности

Попробуйте решить каждую из этих задач, обращая внимание на предыдущий набор задач. Ищите закономерности, чтобы упростить решение задач.

3 × 3 = ?
3 × 2 = ?
3 × 1 = ?
3 × 0 = ?
3 × -1 = ?
3 × -2 = ?
3 × -3 = ?
2 × -3 = ?
1 × -3 = ?
0 × -3 = ?
-1 × -3 = ?
-2 × -3 = ?
-3 × -3 = ?

Ответы на эти задачи приведены ниже, но я настоятельно рекомендую сначала самостоятельно решить приведенные выше задачи, чтобы вы поняли, как учащиеся могут продумать этот набор задач.

3 × 3 = 9
3 × 2 = 6
3 × 1 = 3
3 × 0 = 0

На этом этапе многие люди заметят, что ответы каждый раз становятся на 3 меньше, а число, умножаемое на 3, каждый раз уменьшается на единицу, поэтому они продолжают использовать эту схему, чтобы ответить на следующие вопросы.

3 × -1 = -3
3 × -2 = -6
3 × -3 = -9

Теперь уменьшим первое число в шаблоне на 3, и нужно сделать некоторые выводы о том, что ответ должно быть.

2 × -3 = -6
1 × -3 = -3
0 × -3 = 0

Теперь можно заметить, что ответы увеличиваются на 3 каждый раз, когда мы увеличиваем первое число, и поэтому разумно продолжать эту схему.

-1 × -3 = 3
-2 × -3 = 6
-3 × -3 = 9

Хотя для некоторых эта закономерность может показаться очевидной, когда кто-то еще находится в середине изучения этой концепции, у них есть меньше когнитивных способностей, доступных для выполнения поставленной задачи (умножения чисел) и выполнения дополнительной задачи по поиску закономерностей в их ответах, так что именно здесь кто-то еще побудит их остановиться и поискать закономерности в их работе до сих пор будет очень полезный.

Необходимые знания : Нужно знать, что означают эти символы, что подразумевается под нахождением одного числа, умноженного на другое, и как работают отрицательные числа с точки зрения обратного отсчета и вычитания.

 

Математическая непротиворечивость и математические свойства

Давайте рассмотрим задачу, которую мы можем решить несколькими способами, заимствованными из Академии Хана.

5 × (3 + -3) = ?

Если мы сначала добавим числа в скобках, то это будет 5 умножить на 0, что равно 0, так как 3 + -3 = 0,

5 × (3 + -3) = 0

Но что, если мы сначала распределим 5 через оба слагаемых?

5 × 3 + 5 × -3 = ?

Поскольку распределение 5 по сложению не меняет значение выражения, мы знаем, что оно по-прежнему равно 0.

5 × 3 + 5 × -3 = 0

Но это означает, что 5 × 3 и 5 × -3 противоположные знаки, так как 5 × 3 = 15, то 5 × -3 равно -15. Давайте посмотрим на другой пример.

-5 × (3 + -3) = ?

Мы знаем, что это то же самое, что -5 умножить на 0, так что это значение равно 0,

-5 × (3 + -3) = 0

Как и раньше, мы распределяем -5 через оба слагаемых.

-5 × 3 + -5 × -3 = ?

Опять же, распределение членов не меняет значения выражения в левой части уравнения, поэтому результат по-прежнему равен 0.

-5 × 3 + -5 × -3 = 0

Мы заранее известно, что -5 × 3 равно -15, поэтому мы можем заменить это значение на -5 × 3 в левой части уравнения.

-15 + -5 × -3 = 0

Следовательно, -15 и -5 × -3 являются противоположностями, поскольку они добавляют к 0, поэтому -5 × -3 должно быть положительным.

Ничто из того, что мы сделали для двух приведенных выше примеров, не относится к значению 5 × 3, поэтому мы можем повторить этот аргумент для любого другого факта умножения, который мы хотим вывести, так что эти две идеи можно обобщить.

Необходимые знания : Нужно знать, что означают эти символы, что подразумевается под нахождением одного числа, умноженного на другое, как работает распределительное свойство и как отрицательные числа могут быть определены как противоположные положительным числам.

 

Представление на числовой прямой

Представьте, что мы представляем умножение в виде скачков на числовой прямой.

3 раза по 3 на числовой прямой

Для 3 × 3 мы рисуем 3 группы по 3, двигаясь вправо. Как количество групп, так и направление каждой группы указаны справа.

А как насчет 3 × -3? Теперь у нас есть еще 3 группы числа, но число отрицательное.

3 раза -3 на числовой прямой

Если мы находим -3 × 3, размер и направление умножаемого числа те же, но теперь мы находим -3 группы этого числа. Один из способов подумать об этом — подумать об удалении 3 групп числа. Другой вариант — рассматривать число, умноженное на -3, как отражение 3-кратного одного и того же числа.

-3 умножить на 3 на числовой прямой

Таким образом, -3 × -3 является отражением 3 × -3 на числовой прямой.

-3 умножить на -3 на числовой прямой

Однако в некотором смысле этот визуальный аргумент является просто математической последовательностью, представленной с помощью числовой прямой. Если умножение на отрицательное число является отражением через 0 на числовой прямой, а мы думаем об отрицательных числах как отражение через 0 на числовой прямой, то умножение отрицательного числа на отрицательное число является двойным отражением.

 

Контекст

У Карен Лью есть аналогия.

Умножение на минус — это повторное вычитание. Когда мы умножаем отрицательное число на отрицательное число, мы получаем меньше отрицательного числа.

Эта аналогия между умножением, сложением и вычитанием помогает учащимся красиво связать эти два понятия.

Джозеф Рурк поделился этим контекстом.

Игрок проигрывает 10 долларов в день. Насколько больше денег у них было 5 дней назад?

Здесь убыток за день — это одно отрицательное значение, а движение назад во времени — другое.

@M_Teacher_w_T поделился этой аналогией:

«Враг моего врага — мой друг».

Это направлено не на алгебраические или арифметические свойства чисел, а скорее на противоположность отрицательных чисел.

Необходимые знания: Все контексты, формирующие новое понимание, требуют, чтобы учащиеся достаточно хорошо понимали части контекста, поэтому особенно важно выяснить, как учащиеся понимают идею, когда она представлена ​​в контексте.

 

Алгебраическое доказательство из первых принципов

От доктора Алекса Юстиса мы получили это алгебраическое доказательство того, что отрицание, умноженное на отрицательное, является положительным.

Во-первых, он формулирует набор аксиом, применимых к любому кольцу с единицей. Кольцо — это, по сути, система счисления с двумя операциями. Каждая операция является закрытой, а это означает, что использование этих операций (таких как сложение и умножение действительных чисел) приводит к другому числу в системе счисления. Каждая операция также имеет элемент идентичности или элемент, который не изменяет другой элемент в системе при применении к ней. Например, при добавлении 0 является аддитивной идентичностью. При умножении 1 является мультипликативным тождеством. Полный набор необходимых аксиом приведен ниже.

A × C (LOATE LAPED). : ( B + C ) × A = B × A + C × A
Axiom 1 : a + b = b + a (Additive commutivity)
Axiom 2 : ( a + b ) + c = a + ( b + c ) (Additive associativity)
Axiom 3 : 0 + a = a (Additive identity)
Axiom 4 : существует — A Удовлетворяет A + ( — A ) = 0 (Additial Relverse)
AXIOM 5

: 1 × = = = = = = = = = = . = a
(мультипликативная идентичность)
Axiom 6 : ( A × B ) × C = A × 4544544544544. 415441544. . . . . . 4. ). ассоциативность)
Axiom 7 : A × ( B + C ) = A × B + A × C (правая размножение PROPLACTION). отрицательный раз отрицательный является положительным. Ниже я воспроизведу доказательство доктора Юстиса и включу ссылку на использованные аксиомы. Сначала докажем, что a = -(- a ).

Corrolarary 1

a. a ))
A = A + 0 (Axiom 3 и Axiom 1)
4 A 40189
4 A A
A.
(Axiom 4, применяемый к — A )
A = ( A + ( — A )) + ( — ( — A. ) 9055)) + ( — ( — A ) )) + ( — A

)) + ( — A. ))) + ( A + ( — A )) + ( A + ( — A )) + ( A + ( — A )) + ( A + ( — A )) + ( A + ( — A )). 2 – ассоциативное свойство)
A = 0 + ( — ( — A )) (аксиома 4)
A = — ( A )

). мы знаем, что если ввести отрицательные числа , то будет равно -(-?).

Corrolary 2


9

9

4

0 = A + ( — A ) (AXIOM 4)
0 = (0 + 1)
0 = (0 + 1)
0 = (0 + 1)
0 = (0 + 1)
0 = (0 + 1)
(0 + 1)
(0 + 1)
(0 + 1). (Axiom 3 и Axiom 5)
0 = 0 × A + 1 × A + ( — A ) (AXIOM 8)
(AXIOM 8)
(AXIOM 8)
(AXIOM 8)
(AXIOM 8) (AXIOM 8) (AXIOM 8). ( a + (− a )) (Axiom 5 and Axiom 2)
0 = 0 × a + 0 (Axiom 4)
0 = 0 × a (Аксиома 3 и Аксиома 1)

Доказательство того, что 0 = 0 × a — это до боли очевидная идея, которая едва ли требует доказательств, но она устанавливает связь между умножением и аддитивной идентичностью в действительных числах. который еще не включен в приведенные выше аксиомы.

Далее докажем, что (−1) × a = − a .

Corrollary 3

9017 — + 5 a =
A = — A + 0 × A (Corrolary 2 и Axiom 3)
(Corrolary 2 и Axiom 3)
(Corrolary 2 и Axiom 3). + (−1)) × a (аксиома 4)
a = − a + 1 × a + (−1) ×

4 a 5

a = (− a + a ) + (−1) × a (аксиома 5 и аксиома 2)
× A (Axiom 4)
A = 0 + (−1) × A (Axiom 3)
(AXIOM 3)
(AXIOM 3). a ) × (− b ) = ab .

0314

Это последнее «доказательство» вряд ли оправдывает тот факт, что отрицательный результат, умноженный на отрицательный, является положительным для любого студента. Это то, что является необходимым уровнем обоснования для математика, заинтересованного в строгом доказательстве, который, вероятно, счел бы другие обоснования «шаблонными» и недостаточными.

Критическая идея доказательства, однако, заключается в том, что целевая аудитория доказательства остается убежденной в том, что идея верна, и поэтому я утверждаю, что представленное здесь алгебраическое «доказательство» вообще не является доказательством почти для всех.

Необходимые знания : Пока я прорабатывал и добавлял обоснование для каждого отсутствующего шага доказательства, мне нужно было достаточно бегло обращаться с первоначальным набором аксиом. Мне также нужно было не упускать из виду общую цель и уметь распознавать структуру каждой части аргумента и сопоставлять эту структуру с аксиомами.

 

Более простое алгебраическое доказательство

Это алгебраическое доказательство Бенджамина Дикмана намного проще, чем возвращение к доказательству, основанному на аксиомах арифметики.

A + ( — A ) = 0
A × B + ( — A ) × B = 0 × B
AB = 0 × B
AB + ( B
AB +) ( B ) AB + ( B ). = 0

Отсюда мы можем показать, что ab и – ab имеют противоположные знаки и, следовательно, положительное произведение на отрицательное является отрицательным. Используя факт, умножение коммутативно, отрицательное произведение, умноженное на положительное, также является отрицательным.

Точно так же мы можем доказать, что отрицательное произведение, умноженное на отрицательное, является положительным.

a + (− a ) = 0
a × (− b ) + (− a ) × (− b ) = 0 × (4 − 90 5 ) 0 5 b 3 0 5 (0 − 90) ab + (− a ) × (− b ) = 0

Поскольку мы знаем, что − ab отрицательно, а сумма этих двух членов равна 0, поэтому (− a ) × (− b ) положительный.

Предварительные знания : Необходимые знания для этого доказательства намного меньше, чем для другого, но оно предполагает достаточное владение алгебраическими структурами.

 

Вывод:

Учитывая, что цель аргумента в пользу истинности чего-либо состоит в том, чтобы убедить другого человека в истинности аргумента, всякий раз, когда кто-либо использует какое-либо обоснование, репрезентацию или доказательство, следует убедиться, что аудитория убеждена.

1.5 Почему NEGATIVE TIMES NEGATIVE POSITIVE?

Когда мы обнаруживаем отрицательные числа, мы естественно, даже без сомнения, предполагаем, что они подчиняются тем же законам арифметики, что и обычные положительные счетные числа. То есть нам нравится верить, что основные законы, такие как \(a\times b=b\times a\) и (a\times 1=a\) и \(a \times 0=0\) выполняются для всех числа, как положительные, так и отрицательные, и что мы можем раскрывать скобки даже с отрицательными значениями и так далее. Конечно, эти правила предполагают, что мы знаем априори умеют умножать отрицательные числа.

УМНОЖЕНИЕ ПОЛОЖИТЕЛЬНЫХ И ОТРИЦАТЕЛЬНЫХ ЧИСЕЛ

В начальной учебной программе умножение вводится в контексте целых счетных чисел и определяется там как многократное сложение. Например, \(4 \times 5\) читается как «четыре группы по пять» и вычисляется следующим образом: \(4 \times 5 = 5 + 5 + 5 + 5 = 20\).

На самом деле довольно неожиданно, что \(5 \умножить на 4\), «пять групп по четыре» дает тот же числовой ответ, что и четыре группы по пять. Расчет совсем другой, когда числа 5 и 4 поменялись ролями.

 

Вопрос: Как бы вы убедили молодого студента в том, что 193 группы по 307 обязательно будут иметь такое же значение, как 307 групп по 193? Почему мы хотим верить, что \(a \times b = b \times a\) для подсчета чисел? (СОВЕТ: расставьте точки прямоугольными рядами.)

 

Повторное сложение позволяет нам умножать положительное число и отрицательное число. Например, \(2 \times \left(-3\right)\) может быть прочитано как «две группы отрицательных трех» и поэтому вычисляется как \(2 \times \left(-3\right)=-3 +\;-3=-6\). С помощью свай и отверстий это выглядит так:

Интерпретация отрицательного числа, умноженного на положительное, и отрицательного, умноженного на отрицательное путем многократного сложения, однако, проблематична.

Что может означать \(\left(-2\right) \times 3\) ? «Отрицательные две группы по три» не имеет смысла.

И \(\left(-2\right) \times \left(-3\right)\) одинаково странно: «отрицательные две группы отрицательных трех».

 

Правда в том, что умножение здесь не имеет смысла в контексте многократного сложения. Мы вступили на новую территорию, и если мы хотим открыть наш мир для новых типов чисел, неудивительно, что ранее конкретные, буквальные определения начинают колебаться. Таким образом, мы должны осуществить изощренный сдвиг мышления, отпустив вопрос 9. 0054 Что такое умножение? , чтобы вместо этого спросить:

Как бы мы хотели, чтобы вело себя умножение?  

 

Комментарий: Позвольте мне подчеркнуть этот момент. Вопрос «Что означает умножение отрицательных чисел?» — это вводящий в заблуждение вопрос, и это не тот вопрос, который стоит задавать на данном этапе нашей работы: мы все еще пытаемся решить вопрос о том, чем может быть умножение в мире отрицательных чисел. Чтобы подойти к этому, мы сначала должны четко определить, какие особенности арифметики, по нашему мнению, должны оставаться верными.

 

ДУМАЕМ ЧЕРЕЗ ВЕЩИ

Положительное время Отрицательное :  Кажется убедительным придерживаться понятия «повторяющегося сложения» для произведения отрицательного и положительного:

9002 \(2\002 с \left(-3\right)=\) две группы отрицательных троек \(=-3+-3=-6\).

Большинство людей согласны с тем, что мы должны придерживаться этой идеи.

 

Отрицательное время Положительное : Это проблематично: \(\left(-2\right)\times 3=?\)

Но кажется убедительным утверждение, что коммутативный закон \(a \times b = b\times a\) должен выполняться для всех типов чисел, включая отрицательные числа. В этом случае мы можем написать:

\(\left(-2\right)\times 3 =3\times \left(-2\right)\) три группы отрицательных двух \(=-2+-2+ -2=-6\).

 

Отрицательное время Отрицательное : Как мы должны вычислить \(\влево(-2\вправо) \умножить на \влево(-3\вправо)\)?

Применение коммутативного закона и представление об этом как \(\влево(-3\вправо) \times \влево(-2\вправо)\) в этом случае не помогает. Итак, какая математика могла бы направить нас в нашем мышлении здесь?

Мы говорили, что нам нравится верить, что все обычные законы арифметики ( \(a\times b=b\times a\), \(a \times 1=a\), \(a \times 0= 0\), раскрывающиеся скобки и т. д.) должны выполняться для всех типов чисел. Поскольку модель площади — это просто представление нашей веры в расширение скобок, модель площади должна работать и для отрицательных чисел!

КЛЮЧЕВОЙ ПРИМЕР: Вот три способа вычислить \(17\умножить на 18\), представляя  \(17\) либо как \(10+7\), либо как  \(20+\влево(-3\вправо). )\) и \(18\) как \(10+8\) или \(20 + \влево(-2\вправо)\). Хотя с геометрической точки зрения нет смысла иметь отрицательную длину стороны геометрической фигуры, мы видим, что математика, которую представляет каждая диаграмма, по-прежнему является правильной математикой.

Но есть и четвертая возможная картинка!

Математика раскрывающихся скобок предполагает, что правильное значение \(\left(-2\right)\times\left(-3\right)\) равно \(+6\). (В произведении по-прежнему должен быть ответ \(306\).)

 

УПРАЖНЕНИЕ: Нарисуйте четыре диаграммы, представляющие \(26\x 35\), и используйте последнюю, чтобы продемонстрировать, почему мы должны установить \(\ влево(-4\вправо)\раз\влево(-5\вправо)=+20\).

 

ТОЧНЫЙ ЛОГИЧЕСКИЙ АРГУМЕНТ, ПОЧЕМУ ОТРИЦАТЕЛЬНОЕ ВРЕМЯ ОТРИЦАТЕЛЬНОЕ ДОЛЖНО БЫТЬ ПОЛОЖИТЕЛЬНЫМ ) и \(\left(-3\right)\times 2=-6\) (через веру в коммутативность), что отрицательное, умноженное на отрицательное, является положительным, является вынужденным логическим следствием следующих двух основных убеждений арифметики: \( a\times 0 = 0\) и \(a\left(b+c\right)=ab+ac\). Вот почему:

 

Докажем \(\влево(-2\вправо)\раз\влево(-3\вправо)=+6\).

 

По первому из правил мы должны сказать: \(\left(-2\right)\times 0 = 0\).

Переписав первый ноль, мы должны договориться, что: \(\влево(-2\вправо)\times \влево(3+\влево(-3\вправо)\вправо) = 0\).

При распределении мы также должны согласиться с тем, что: \(\влево(-2\вправо)\умножить на 3 + \влево(-2\направо)\раз\влево(-3\направо)=0\).

Это выглядит так: \(-6 + \влево(-2\вправо)\times\влево(-3\вправо)=0\).

Отсюда следует, что \(\влево(-2\вправо)\times\влево(-3\вправо)\) должно быть \(+6\).

 

УПРАЖНЕНИЕ: Создайте аналогичный аргумент, чтобы установить, что \(\left(-4\right)\times\left(-5\right)=+20\).

 

Итак… ЧТО ТАКОЕ УМНОЖЕНИЕ?

 

В контексте положительных целых чисел повторяется сложение.

 

В контексте положительных и отрицательных целых чисел я лично не знаю, что это такое, кроме математически согласованного набора операций, так что если \(a\) и \(b\) — положительные целые числа, тогда \(a\times \left(-b\right)=-ab\) и с логическими последствиями \(\left(-a\right)\times b = b \times \left(-a\right) =-ab\) и \(\left(-a\right)\times\left(-b\right) = ab\).

 

Люди пытаются придать всему этому конкретный смысл с моделями солдат, марширующих по числовым линиям, поворачивающих в разные стороны, системами прибыли и долга, работающими при плюсовых и минусовых температурах и так далее. Каждая модель хороша для иллюстрации НЕКОТОРЫХ аспектов арифметики отрицательных чисел, но не всех. Например, идея «убрать пять градусов холода — это то же самое, что добавить пять градусов тепла» может помочь некоторым объяснить, почему \(-\left(-5\right)\) должно равняться \(5\) , но само по себе это не объясняет, почему отрицательное, умноженное на отрицательное, называется положительным.

 

С педагогической точки зрения нам нужно отойти от того, чтобы ученик начал думать об умножении отрицательных чисел с помощью моделей, которые пытаются, но должны в какой-то момент потерпеть неудачу, «объяснить», что такое умножение отрицательных чисел. Вместо этого мы должны начать с обсуждения того, что, по нашему мнению, должно быть верным в отношении умножения в целом и как оно ведет себя. Раскрытие скобок с помощью модели площадей дает убедительную студенческую иллюстрацию того, что математика «хочет», чтобы отрицательные числа, умноженные на отрицательные, были положительными. (А для студентов, готовых к этому, аксиоматический подход завершает это.) Что на самом деле означает выражение «отрицательное, умноженное на отрицательное, становится положительным» — я понятия не имею. Я просто знаю, что это алгебраически непротиворечиво.

 

УПРАЖНЕНИЕ:  а) Докажите, что \(-a\) и \(\left(-1\right) \times a\) — одно и то же число. (СОВЕТ: \(a+\left(-1\right)\times a = 1\times a + \left(-1\right)\times a = \ldots\) .)

b) Если вы верите что \(-\left(-5\right) = 5\) объясняют, почему теперь следует, что \(\left(-1\right) \times \left(-1\right) = 1\).

 

Присоединяйтесь к обсуждению в Facebook и Twitter и поделитесь этой страницей, используя кнопки ниже.

Facebook

Twitter

абстрактная алгебра — Почему отрицательное умножение на отрицательное = положительное?

$\begingroup$

Это довольно мягко, но однажды я видел аналогию в Интернете, чтобы объяснить это.

Если вы снимаете человека, бегущего вперед ($+$), а затем воспроизводите фильм вперед ($+$), он все еще бежит вперед ($+$). Если вы воспроизведете фильм в обратном направлении ($-$), он покажется бегущим в обратном направлении ($-$), поэтому результат умножения положительного и отрицательного будет отрицательным. То же самое происходит, если вы снимаете человека, бегущего задом наперед ($-$) и воспроизводите его в обычном режиме ($+$), кажется, что он все еще бежит назад ($-$). Теперь, если вы снимаете человека, бегущего назад ($-$), и воспроизводите его в обратном направлении ($-$), кажется, что он бежит вперед ($+$). Уровень, до которого вы ускоряете перемотку, не имеет значения ($-3x$ или $-4x$), эти результаты остаются верными. $$\text{назад} \times \text{назад} = \text{вперед}$$ $$ \text{отрицательный} \times \text{отрицательный} = \text{положительный}$$ Это не идеально, но вводит понятие числовой прямой, по крайней мере, имеющей направления.

$\endgroup$

4

$\begingroup$

Неформальное обоснование $\text{positive} \times \text{negative} = \text{negative}$

Продолжите шаблон:

$$ \начать{массив}{г} 2 & \times & 3 & = & 6\\ 2 & \раз & 2 & = & 4\\ 2 & \times & 1 & = & 2\\ 2 & \times & 0 & = & 0\\ 2 & \times & -1 & = & ? & (\текст{Ответ} = -2 )\\ 2 & \times & -2 & = & ? & (\текст{Ответ} = -4)\\ 2 & \times & -3 & = & ? & (\текст{Ответ} = -6 )\\ \конец{массив} $$

Число в правой части продолжает уменьшаться на 2.


Неформальное обоснование $\text{negative} \times \text{negative} = \text{positive}$

Продолжите шаблон:

$ $ \начать{массив}{г} 2 & \times & -3 & = & -6\\ 1 & \times & -3 & = & -3\\ 0 & \times & -3 & = & 0\\ -1 & \times & -3 & = & ? & (\текст{Ответ} = 3 )\\ -2 & \times & -3 & = & ? & (\текст{Ответ} = 6 )\\ -3 & \times & -3 & = & ? & (\текст{Ответ} = 9)\\ \конец{массив} $$

Число справа постоянно увеличивается на 3.

$\endgroup$

9

$\begingroup$

Что ж, если бы я мог объяснить это кому-то интуитивно (или, по крайней мере, попытаться), я хотел бы провести аналогию с ходьбой по реальной линии, согласившись, что ходьба налево будет идти в отрицательном направлении и идти прямо в положительном направлении.

Затем я попытаюсь передать идею о том, что если вы умножаете два числа (предположим, что они целые, чтобы было проще представить), то произведение $2*3$ просто означает, что вы должны идти прямо (в положительном направлении) расстояние в 2$ (например, мили) три раза, то есть сначала вы проходите 2$ мили, затем еще 2$ мили и, наконец, еще 2$ мили вправо.

Теперь вы представляете, где вы находитесь? Ну, вы справа от начала координат, так что вы находитесь в положительной части. Но точно так же вы можете обыграть эту идею с негативом, умноженным на позитив.

Имея в виду тот же пример, что будет означать $-2*3$? Во-первых, предположим, что $-2$ просто указывает, что вам придется пройти налево расстояние в $2$ миль. Тогда сколько раз вы пройдете это расстояние? Так же, как и раньше, раз на $3$, и в конце концов вы окажетесь на $6$ миль левее исходной точки, так что вы окажетесь в отрицательной секции.

Наконец, вам нужно попытаться представить, что может означать $(-2)*(-3)$. Может быть, вы могли бы подумать о отрицательном знаке во втором факторе, означающем, что вы меняете направление, то есть заставляет вас развернуться и начать идти указанное расстояние. Таким образом, в этом случае $-2$ говорит вам пройти налево расстояние в $2$ миль, но $-3$ говорит вам сначала развернуться, а затем пройти $3$, умноженные на $2$ миль, в другом направлении, так что вы в конечном итоге пойдете вправо и закончите в точке, которая находится в $6$ милях справа от начала координат, так что вы окажетесь в положительном сечении, и $(-2)*(-3) = 6$.

Я не знаю, поможет ли это, но это единственный способ, которым я могу думать об этом в каком-то интуитивном смысле.

$\endgroup$

1

$\begingroup$

Кто-то недавно прислал мне это:

Я даю вам три купюры по 20 долларов: +3 * +20 = +\$60 для вас
Я даю вам три долга по 20 долларов: +3 * -20 = -\60 долларов для вас
Я беру от вас три купюры по 20 долларов: -3 * +20 = -\$60 для вас 9{\Large =\,0})(\color{#c00}{-y}) + xy = xy$

Эквивалентно, $ $ оценить $\rm\,\ \overline{(-x)(-y) \ +\ } \overline{ \underline {\color{#c00}{x(-y)}}}\underline{\phantom{(}\! +\,\color{#0a0}{xy}}\, $ в $\:\!2\:\!$ способами (обратите внимание на подчеркнутые термины $ = 0)$

Сказано больше концептуально $\rm (-x)(-y)\ $ и $\rm\ :\color{#0a0}{xy}\:$ являются аддитивными инверсиями $\rm\ \color{#c00}{x(-y)}\ $, поэтому они равны в силу единственности инверсий: $ $ т. е. если $\rm\,\color{#c00}a\,$ имеет две аддитивные обратные $\rm\,{-a}\,$ и $\rm\,\color{#0a0}{-a}, \,$ затем 9{\ large = \, 0} + \ color {# 0a0} {-a} \, = \, \ color {# 0a0} {-a} \ qquad $ $

Эквивалентно, $ $ оценить $ \ rm \ ,\ \overline{-a\, +\!\!} \overline{\phantom{+}\! \underline {\color{#c00}{a}}}\underline{\ + \color{#0a0}{-a}}\ $ $\,2\,$ способами (обратите внимание на подчеркнутые термины $ = 0 )$

В этом доказательстве Закона Знаков используются хорошо известные законы натуральных чисел (особенно распределительный закон ), поэтому, если мы потребуем, чтобы эти законы сохранялись в других «числовых» системах, то Закон Знаков является логическим следствием этих основных законов (абстрагированных от законов (положительных) целых чисел).

Эти фундаментальные законы «чисел» аксиоматизированы алгебраической структурой, известной как кольцо, и различными ее специализациями. Поскольку приведенное выше доказательство использует только кольцевых закона (в первую очередь дистрибутивного закона ), закон знаков выполняется в каждом кольце, например. кольца полиномов, степенные ряды, матрицы, дифференциальные операторы и т. д. На самом деле каждая нетривиальная кольцевая теорема (т. е. та, которая не вырождается в теорему о лежащей в основе аддитивной группе или мультипликативном моноиде) должна использовать дистрибутивный закон, поскольку это единственный закон, который связывает аддитивную и мультипликативную структуры, которые в совокупности образуют кольцевую структуру. Без дистрибутивного закона кольцо вырождается в множество с двумя совершенно не связанными между собой аддитивной и мультипликативной структурами. Итак, в каком-то смысле распределительный закон — это краеугольный камень кольцевой структуры.

Замечание $\ $ В более общем случае Закон знаков выполняется для любых нечетных функций под композицией, т.е. многочлены, все члены которых имеют нечетную степень. Действительно, у нас есть

$$\begin{align}\rm f(g)\ =\ (-f)\ (-g)\ =\:\! -(f(-g)) \iff\,&\rm \ f(-g)\ = -(f(g))\\ \rm \overset{ \large g(x)\,=\,x}\iff&\rm \ f(-x)\ = -f(x),\ \ \text{т. е. $\rm\:f\:$ нечетно} \end{align}\qquad$$

Обычно такие функции имеют только более слабую рядом с — кольцевая структура. В приведенном выше случае колец из дистрибутивности следует, что умножение линейно, следовательно, нечетно (рассматривая кольцо в стиле Кэли как кольцо эндоморфизмов своей абелевой аддитивной группы, т. е. представление каждого кольцевого элемента $\rm\r\$ линейным отображением $\rm\x\to r\x,\$ т. е. как $1$-мерная матрица).

$\endgroup$

5

$\begingroup$

Я думаю, что многие ответы либо слишком просты, либо слишком сильно отклоняются от математики. Просто помните, что умножение — это многократное сложение. При работе с отрицательными числами это становится повторным вычитанием.

Я бы просто сказал в этом контексте:

  1. Уравнение: $$\begin{уравнение*}\begin{массив}{c} \фантом{\times9}2\\ \подчеркнуть{\раз\фантом{9}2}\\ \фантом{\times9}4\\ \end{массив}\end{уравнение*}$$ просто добавляет положительный $ 2 $, два раза.

  2. Уравнение: $$\begin{уравнение*}\begin{массив}{c} \фантом{\times999}2\\ \подчеркнуть{\раз\фантом{1}-2}\\ \фантом{\times9}-4\\ \end{массив}\end{уравнение*}$$ это просто добавление положительных $2$, отрицательных два раза, что означает, что вместо добавления в положительном направлении вы добавляете в отрицательном направлении (вычитание).

    Вы также можете просто сказать, что добавляете $-2$ (или вычитаете $2$) два раза.

  3. Уравнение: $$\begin{уравнение*}\begin{массив}{c} \фантом{\times9}-2\\ \подчеркнуть{\раз\фантом{1}-2}\\ \фантом{\times999}4\\ \end{массив}\end{уравнение*}$$ добавляет $-2$, минус два раза. Добавление $-2$ два раза дает диаграмму в (2). Поскольку вам нужно сложить с минусом два раза, вы измените направление сложения.

    Можно также сказать, что вы вычитаете $-2$ два раза.

$\endgroup$

5

$\begingroup$

Простой ответ: $$ (-а)б + аб = (-а)б + аб $$ $$(-a)b + ab = b(a-a) $$ $$(-a)b + ab = b(0) $$ $$(-a)b + ab = 0 $$ $$(-a)b = -ab $$ $$(-a)(-b) + (-ab) = (-a)(-b) + (-a)b $$ $$(-a)(-b) + (-ab) = (-a)(b-b) $$ $$(-a)(-b) + (-ab) = (-a)(0) $$ $$(-a)(-b) + (-ab) = 0 $$ $$*(-a)(-b) = аб $$ Надеюсь, это поможет (Кредит исчисления Майкла Спивака)

~ Алан

$\endgroup$

3

$\begingroup$

Это действительно один из тех важных вопросов, которые заставляют многих людей говорить: «Математика — отстой!». На самом деле, для многих студентов математика потеряла смысл где-то в процессе. Медленно или драматично они сдавались на поле как безнадежно непонятном и сложном и вырастали взрослыми, которые, уверенные в том, что другие разделяют их опыт, небрежно заявляют: « Математика была просто не для меня » или « Я никогда не был хорош в этом. ” Обычно процесс идет постепенно, но для Рута Макнейл, поворотный момент был четко определен. В статью в Journal of Mathematical Behavior, она описал как это было:

Меня привлекла идея, что отрицательное число раз отрицательное число превращается в положительное количество. Это казалось (и до сих пор кажется) по своей сути маловероятно — контринтуитивно, как говорят математики. Я боролся с идеей того, что я себе представляю несколько недель, пытаясь получить разумное объяснение от моего учителя, моих одноклассников, моих родителей, любого- тело. Какие бы объяснения они ни предлагали, они не могли преодолеть мое сильное чувство, что умножение что-то усиливает, и, таким образом, два отрицательных числа умноженные вместе должны должным образом давать очень отрицательный результат. С тех пор мне предложили умеренно убедительное объяснение, в котором фигурирует фильм бассейн осушается, что возвращается назад- палаты через проектор. Однако в то время меня ничего не убедило. Самый здравый смысл все школьные предметы отказались от здравого смысла; Я был возмущен и сбит с толку.

Тем временем учебная программа продолжалась, и я мог видеть, что я не могу оставаться позади, застряв на отрицательном тивные времена отрицательные. я бы обратил внимание на следующая тема и единственный практический курс, открытый для Мне нужно было притвориться, что согласен с тем, что отрицательные времена отрицательные. тивное равно положительному. Книга и учитель и общее мнение выживших после алгебры так- общество было явно более могущественным, чем я. я капито- поздно Я сделал остальную часть алгебры, и геометрии, и тригонометрия; Я делал их в продвинутых разделах, и у меня часто возникало приятное чувство «ага!» когда я мог внезапно увидеть, как придет доказательство вне. Но под ней какая-то обида и затаилось предательство, и я не удивился и не встревожился из-за дальнейших глупостей моих учителей математики рукава… Интеллектуально я был свободен, и когда математика стала не нужна, я взял немецкий вместо.


В этом ответе я покажу, что: отрицательный $\times$ отрицательный $=$ положительный на самом деле совсем не противоречит здравому смыслу! Есть много способов, которые мы можем использовать, чтобы показать этот результат, но я хотел бы показать мой личный способ думать о последнем.

Представим, что мы сидим у дороги, а рядом с ней движется автомобиль с постоянной скоростью. У нас также есть часы, и поэтому мы можем измерять время.

Прежде чем двигаться дальше, мы должны сначала указать некоторые предположения, например, если автомобиль движется вправо, то его скорость будет положительной, а если он движется в левом направлении, то его скорость будет отрицательной.

Теперь представьте, что у вас есть видео с показанной выше сценой, и время положительно, если вы воспроизводите видео в обычном режиме, но отрицательно, если вы воспроизводите его в обратном порядке. Мы также знаем следующее: $$\rm Velocity=\dfrac{\rm Distance}{\rm Time}.$$ Находя расстояние, получаем: $$\rm Velocity\times{\rm Time}={\rm Distance}.$$ Здесь наступает важная часть, если машина движется в направлении $+$ и время воспроизведения видео положительное, т.е. видео воспроизводится нормально, то вы увидите, что машина движется в направлении $+$ и вы подсчитаете, что он перемещается на «положительное расстояние». Поэтому верно следующее: $$\rm положительный\times положительный=положительный.$$

Если машина движется в направлении $-$ и время воспроизведения видео положительное, т.е. видео воспроизводится нормально, то вы увидите, что машина движется в направлении $-$, тогда вы посчитаете, что он перемещается на «отрицательное расстояние». Таким образом: $$\rm отрицательное\раз положительное=отрицательное.$$

Если же автомобиль движется в направлении $+$, но время воспроизведения видео отрицательное, т.е. видео воспроизводится в обратном направлении, то вы увидите, что машина движется в направлении $-$, и вы подсчитаете, что она перемещается на «отрицательное расстояние». Таким образом: $$\rm положительное\times отрицательное=отрицательное.$$

Если машина движется в направлении $-$, но время воспроизведения видео отрицательное, т.е. видео воспроизводится в обратном направлении, вы увидите, что машина движется в направлении $+$! Таким образом, он перемещается на «положительное расстояние». И, следовательно,: }}}$$ Как вы уже убедились, нужно лишь немного воображения, чтобы понять смысл.

Надеюсь, это поможет.
С наилучшими пожеланиями, $\mathcal H$akim.

$\endgroup$

3

$\begingroup$

Элементарную интуицию, стоящую за произведением двух отрицаний, можно представить следующим образом. У вас есть банковский счет. Вы оплачиваете 3 счета по 40 долларов каждый, на ваш счет зачисляется 3$ \cdot (-40) = -120$.

Противоположностью выставления счетов будет выставление счетов кому-то другому.

Итак, если вы выставите счет трем людям по 40 долларов каждому, на ваш счет будет добавлено $(-3) \cdot (-40)$. Это значение должно быть положительным, поскольку в результате вы получаете деньги.

$\endgroup$

$\begingroup$

Вот доказательство. Во-первых, для всех $x$ $x\cdot 0=x\cdot(0+0)=x\cdot 0 +x\cdot 0$. Вычитая $x\cdot0$ с каждой стороны, $x\cdot0=0$. Теперь для всех $x$ и $y$ $0=x\cdot0=x\cdot(-y+y)=x\cdot(-y)+x\cdot y$. Вычитая $x\cdot y$ с обеих сторон, $x\cdot(-y)=-(x\cdot y)$. Применяя это дважды вместе с тождеством $-(-a)=a$, $(-x)\cdot(-y)=-(-x)\cdot y=-(-(x\cdot y))=x \cdot у$.

Ваше доказательство неявно использует тот факт, что $-xy=(-x)y$, и предполагает, что есть только две возможности, $xy$ или $-xy$, а затем показывает, что последнее невозможно. Это кажется правдоподобным предположением, но я старался быть очень осторожным в своем доказательстве выше (таким образом, используя $-(x\cdot y)$, а не просто $-xy$, чтобы не путать с $(-x)\cdot y$ ).

У меня есть только смутное интуитивное представление, которое я, вероятно, не могу хорошо объяснить, но я иногда думаю об отрицательном числе, таком как $-5$, как о «$5$ в другом направлении», и поэтому умножаю на $-5$ означает «умножить на $5$ и изменить направление», т. е. знак. Это означает, что если вы умножаете $-5$ на отрицательное число, вы должны изменить его направление обратно на положительное.

$\endgroup$

0

$\begingroup$

Я всегда рассматривал отрицательные числа как «переворот» на числовой прямой.

Например, -2 то же самое, что и 2, но отражено с другой стороны от нуля.

Умножение тогда работает следующим образом:

2 x 3 не имеет переворотов, так что это просто 2×3 = 6.

-2 x 3 имеет один переворот, поэтому вы начинаете с 2×3 = 6, но с одним переворотом, так что получается -6 вместо.

2 x -3 то же самое, только один переворот, поэтому 2×3 = 6, но перевернутый до -6.

-2 x -3 имеет два переворота, поэтому вы начинаете с 2×3 = 6. Когда вы применяете два переворота, вы возвращаетесь к тому, с чего начали, потому что вы переворачиваете на отрицательное значение, а затем переворачиваете обратно. Значит -3 х -2 = 6,

$\endgroup$

$\begingroup$

Довольно хорошее объяснение состоит в том, что кто-то хочет, чтобы закон распределения работал вообще с положительными величинами, когда вы добавляете (меньшие) отрицательные величины: Если $x>a\ge0$ и $y>b\ge0$, то $$ (xa)(yb)=(x+(-a))(y+(-b))=xy+(-a)y+x(-b)+(-a)(-b) $$ Чтобы это всегда работало, нужно $(-a)y=-(ay)$ в случае $b=0$, $x(-b)=-xb$ в случае $a=0$ и $(- а)(-б)=аб$.

$\endgroup$

$\begingroup$

Я думаю, что x и y немного мешают; вы можете увидеть важные шаги, используя только 1 и -1. На самом деле вы показали, что (-1)(-1)=-1 приводит к противоречию. Если мы разделим на -1, то получим -1=1, что неверно!

Чтобы получить правильный ответ (-1)(-1)=1, нужно сделать еще пару шагов: Во-первых, вы должны согласиться с тем, что (1)+(-1)=0, (1)(-1)=- 1 и (0)(-1)=0.

Теперь умножим первое уравнение на (-1) и воспользуемся свойством распределения, чтобы получить (-1)(-1)+(-1)(1)=(-1)(0). Теперь упростим известные нам части, чтобы получить (-1)(-1)+(-1)=0. Решите для (-1)(-1), и вы получите (-1)(-1)=1.

Итак, мы должны иметь (-1)(-1)=1, если мы принимаем основные правила арифметики: 0 — аддитивная единица, 1 — мультипликативная единица, -1 — аддитивная обратная единице, а умножение распределяется по добавление.

Одним из физических объяснений отрицательного * отрицательного = положительного, которое людям часто нравится, является коэффициент умножения. Вы можете снимать человека, идущего вперед (положительный показатель) или задом наперёд (отрицательный показатель). Теперь воспроизведите фильм, но в обратном порядке (еще одна отрицательная скорость). Что вы увидите, если воспроизведете фильм о том, как кто-то идет задом наперёд? Вы видите человека, идущего вперед, потому что отрицательно*отрицательно=положительно!

$\endgroup$

$\begingroup$

Расширение действительных чисел до комплексной плоскости. Умножение на $-1$ — это поворот на $\pi$ радиан. Когда вы умножаете на два минуса, вы поворачиваете на $2\pi$. 🙂

$\endgroup$

$\begingroup$

Необходимо понимать, что этот закон нельзя доказать так же, как можно доказать законы положительной рациональной и интегральной арифметики. Причина этого в том, что у негативов нет никакого «внешнего» (внешнего по отношению к математике, т.е. доаксиоматическое, интуитивное, концептуальное, эмпирическое, физическое, и т. д. ) определение.

Например. Даже не вдаваясь в аксиомы Пеано, я могу доказать, что, где $a$ и $b$ — натуральные числа, $ab=ba$. Действительно, $ab$ — это просто процесс взятия $a$ наборов из $b$. Возьмем по одному элементу из каждого из этих наборов, сформировав таким образом набор из $a$ элементов. Повторите это $b$ раз: вы явно израсходуете ровно все элементы и получите $b$ наборов из $a$ элементов, другими словами, $ba$. Подобные неформальные (но вполне убедительные, разумные и, я бы сказал, неопровержимые) рассуждения можно использовать, скажем, для демонстрации правил обращения с положительными дробями.

Обратите внимание, что в приведенном выше абзаце я использовал тот факт, что как положительные целые числа, так и положительное целочисленное умножение имеют доаксиоматическое, «физическое» определение.

Спросите кого-нибудь, почему произведение двух отрицательных чисел положительно, и лучшее, что они могут сделать, это объяснить , а не доказать . «Ну, отрицательный вид означает «противоположное», поэтому сделать противоположное дважды означает сделать обычное, то есть положительное» не является доказательством, а просто объяснением, служащим для того, чтобы сделать общепринятую математическую аксиому менее удивительной. Другая распространенная начинается со слов «мы хотели бы, чтобы обычные свойства арифметики сохранялись, поэтому предположим, что они выполняются…», но затем остается объяснить, почему так важно, чтобы выполнялись обычные законы арифметики. Сам Эйлер в одной из первых глав своего учебника по алгебре дал следующие в высшей степени сомнительное обоснование. Обосновав $(-a)b=-(ab)$ аналогиями с долгами, он пишет:

Осталось решить случай, в котором — умножается на -; или, например, -a на -b. Что касается букв, на первый взгляд очевидно, что произведение будет ab; но сомнительно, чтобы знак + или знак — был поставлен перед ним, все, что мы знаем, это то, что это должен быть один или другой из этих знаков. Теперь я говорю, что это не может быть знак -: ибо -а на +b дает -ab, а -а на -b не может дать того же результата, что -а на +b…

При всем уважении к Эйлеру (особенно с учетом того, что это было задумано как вводный учебник), я думаю, мы можем согласиться с тем, что это довольно сомнительный с философской точки зрения аргумент.

Причина, по которой это невозможно, заключается в том, что не существует доаксиоматического определения того, чем на самом деле является отрицательное число или отрицательное умножение. О, вы, вероятно, могли бы придумать определение, включающее противоположные «направления» и понятия симметрии, но это было бы совершенно искусственным и вовсе не очевидно «лучшим» определением. На мой взгляд, негативы в конечном счете лучше всего понимать как чисто абстрактные объекты. Так уж получилось — и это весьма загадочно — что эти совершенно абстрактные законы расчета приводят к физически значимым результатам. Это было прекрасно выражено в 1778 году математиком Джоном Плейфером, когда он обращался к тогдашним спорным вопросам об отрицательных и комплексных числах:0003

Вот парадокс, который еще предстоит объяснить. Если операции этой воображаемой арифметики непонятны, то почему они не совсем бесполезны? Является ли исследование настолько механическим искусством, что его можно проводить с помощью определенных ручных операций? Или истина так легко открывается, что интеллект не нужен для успеха наших исследований?

Цитируется по книге «Отрицательная математика: как математические правила могут быть положительно изогнуты» Альберто А. Мартинеса.

Один из способов решения проблемы состоит в том, что отрицательных числа — это другое название для вычитания . Различия между вычитанием и сложением вынуждают нас, если мы отвергаем отрицания, создавать множество различных правил, охватывающих все различные возможности ($a — b$, $b — a$ и $a + b$, и если конкретная теорема или проблема включает более двух переменных, сложность усугубляется еще больше…). Идею отрицаний можно описать как понимание того, что вместо двух операций и одного типа числа у нас может быть одна операция и два вида числа . В самом деле, если вы начнете с некоторых совершенно физически значимых аксиом о вычитании, вы обнаружите, что закон $(-1)(-1)=1$, по-видимому, подразумевается в них. Подсказка: исходя из очень разумных аксиом $a(b-c)=ab-ac\ ,\ a — (b — c) = a — b + c$, рассмотрим произведение $(a-b)(c-d)$.

Но и это объяснение меня не вполне удовлетворяет. Я пришел к убеждению, что мое образование обмануло меня в том, насколько глубока идея отрицательных чисел, и я ожидаю, что они останутся озадаченными еще много лет. В любом случае, я надеюсь, что что-то из вышеперечисленного будет кому-то полезно.

$\endgroup$

3

$\begingroup$

Возможно, можно получить некоторую интуицию, нанеся на числовую прямую положение каждого числа. Инверсия любого числа визуализируется путем зеркального отображения исходного графика. Таким образом, обратным положительному числу (точке справа от нуля) является отрицательное число (точка слева от нуля на том же расстоянии от нуля). Точно так же обратное отрицательному числу является положительным числом. Если мы согласимся с тем, что умножение числа на -1 — это то же самое, что и нахождение обратного числа, то мы увидим, что произведение двух отрицательных чисел должно быть положительным, потому что зеркальное отражение зеркального изображения — это исходное изображение. 9{-1}а\cточка б$$ $$(1)а\cdot б$$ $$=a\cdot b$$

$\endgroup$

3

$\begingroup$

Я предпочитаю объяснение моего любимого математика В. И. Арнольда (на самом деле физика, поскольку, по его собственным словам, «математика есть часть физики» и «экспериментальная наука»). Я считаю, что это наиболее естественное (но полностью математическое) объяснение такого базового понятия, как умножение отрицательных чисел.

Это отрывок из замечательных мемуаров Арнольда «Вчера и давно» (3-е изд. , доступно на английском языке в Springer), полных всемирной истории, драмы и остроумного повествования. Перевод 2007 года на английский, я считаю, не самого лучшего качества, но пока единственный.

из рассказа Семья Арнольдов

Я столкнулся с настоящей трудностью со школьной математикой через несколько лет после таблицу умножения: нужно было выучить, что «минус умножить на минус будет плюс» Я хотел знать доказательство этого правила; Я никогда не мог выучить наизусть то, что неправильно понял. Я попросил отца объяснить, почему (—1) • (—2) = (+2). Он, будучи учеником великих алгебраистов С. О. Шатуновского и Э. Нётер, дал следующее «научное объяснение»: «Точка — сказал он, — что числа образуют поле такое, что дистрибутив закон (x+y)z=xz+yz выполняется. А если бы произведение минус на минус не имело был бы плюс, этот закон был бы нарушен».

Однако для меня это «дедуктивное» (фактически юридическое) объяснение ничего не докажешь — ну и что! Можно изучать любые аксиомы! С в этот день я сохранил здоровое отвращение натуралиста к аксиоматический метод с его немотивированными аксиомами.

Аксиомофил Рене Декарт утверждал, что «ни экспериментальные проверки что аксиомы отражают реальность, ни сравнение теоретических результатов с реальностью должны быть частью науки» (почему результаты должны соответствуют действительности, если исходные принципы не соответствуют Это?).

Еще один тезис декартовской теории и методов воспитания даже своеобразнее и современнее: «Надо запретить все прочие методы обучения кроме моего потому что только этот метод политкорректно : с моим чисто дедуктивным методом любой скучный ученик может обучаться так же успешно, как и самый одаренный , при этом с другими методами воображение и даже рисунки используются неизбежно, и по этой причине гении продвигаются быстрее чем балбесы ».

Вопреки дедуктивным теориям моего отца и Декарта, как десять лет, я начал думать о естественно-научном смысле правила знаков , и я пришел к следующему выводу. А действительное (положительное или отрицательное) число есть вектор на оси координаты (если число положительное, соответствующий вектор положительно направлена ​​вдоль этой оси).

Произведение двух чисел равно площади прямоугольника, стороны которого соответствуют этим числам (один вектор вдоль одной оси и другой вдоль перпендикулярной оси в плоскости). Дан прямоугольник упорядоченной парой векторов, обладает как часть плоскости определенная ориентация (поворот от одного вектора к другому может по часовой или против часовой стрелки). Вращение против часовой стрелки есть обычно считается положительным, и тогда вращение по часовой стрелке отрицательный. И, наконец, площадь параллелограмма (например, прямоугольник), порожденный двумя векторами х и х (снято в определенного порядка), исходящих из одной точки плоскости, считается быть положительным , если пара векторов (взятых в этом порядке) определяет положительная ориентация плоскости (и отрицательная, если поворот от направления вектора х к направлению вектора у отрицательный).

Таким образом, правило знаков — это не аксиома, взятая на ровном месте, а становится естественным свойством ориентации что легко проверить экспериментально.

из рассказа Аксиоматический метод

Моя первая проблема в школе была вызвана правилом умножения отрицательных чисел, и я попросил отца объяснить это своеобразное правило.

Он, как верный ученик Эмми Нётер (и, следовательно, Гильберта и Дедекинд) начал объяснять своему одиннадцатилетнему сыну принципы аксиоматической науки: определение выбирается таким, чтобы распределительная идентичность a(b+c)=ab+ac сохраняется.

Аксиоматический метод требует принятия любой аксиомы с надеяться, что его следствия будут плодотворными (вероятно, это может быть понял к тридцати годам, когда можно было бы читать и оцените «Анну Каренину»). Отец тоже ни слова не сказал о ориентированная площадь прямоугольника или около любой нематематической интерпретация знаков и продуктов.

Это «алгебраическое» объяснение не могло поколебать ни моего сердечного любовь к отцу или глубокое уважение к его науке. Но с тех пор время мне не нравился аксиоматический метод с его немотивированным определения. Вероятно, именно по этой причине к этому времени я получил привыкли разговаривать с неалгебраистами (вроде Л. И. Мандельштама, И. Тамм, П.С. Новиков, Э.Л. Файнберг, М.А. Леонтович, А.Г. Гурвич), которые относились к несведущему собеседнику с полным уважением и пытались объяснить нетривиальные идеи и факты различных наук, таких как как физика и биология, астрономия и радиолокация.

Отрицательные числа я понял год спустя, когда выводил «уравнение времени», учитывающее поправку на продолжительность суток, соответствующая времени года. Это невозможно объяснить алгебраистам, что их аксиоматический метод в основном бесполезна для студентов.

Надо спросить у детей: во сколько завтра будет прилив, если сегодня в 15:00? Это посильная задача, и она помогает детям понимать отрицательные числа лучше, чем алгебраические правила. Однажды я читал у античного автора (вероятно, у Геродота), что приливы «всегда бывают три и девять часов». Чтобы понять, что месячный вращение Луны вокруг Земли влияет на график приливов и отливов. нет необходимости жить рядом с океаном. Здесь не в аксиомах заложена истина математика.

$\endgroup$

4

$\begingroup$

Почему бы нам не рассказать историю ! Банда подлых Далтонов на свободе, но Эл Катчем идет по их следу и чуть не настигает их во время последнего ограбления. При выезде с парковки есть несколько вариантов:

  • Эл быстр, справляется со своей задачей и ловит дальтоны,
  • Машина Ала глохнет, и Далтоны убегают,
  • Далтонам рано сообщают об этом и они убегают, или
  • Машины Далтона глохнут, они захвачены, а город спасен.

Итого:

  • положительный $\times$ положительный: Если хорошая вещь случается с хорошим человеком , это хорошо! 🙂
  • отрицательный $\times$ положительный: Если плохая вещь случается с хорошим человеком , это плохо. 🙁
  • положительный $\times$ отрицательный: Если хорошая вещь случается с плохим человеком , это тоже плохо. 🙁
  • отрицательный $\times$ отрицательный: Но если плохая вещь случается с плохим человеком , это хорошо !! 🙂

$\endgroup$

11

$\begingroup$

Я бы объяснил это цифрами.

Во-первых, чтобы установить, что положительное, умноженное на отрицательное, равно отрицательному: $3 \times 2 = 6, 3 \times 1 = 3, 3 \times 0 = 0$. Обратите внимание, что в каждом случае, когда мы уменьшаем второй фактор на 1, продукт уменьшается на 3. Таким образом, для согласованности следующий продукт в шаблоне должен быть $0 — 3 = -3$. Следовательно, у нас есть $3 \times (-1) = -3, 3 \times (-2) = -6$, а также отрицательное значение для любого другого положительного числа, умноженное на отрицательное.

Во-вторых, чтобы установить, что отрицательное, умноженное на отрицательное, является положительным: теперь мы знаем, что $3 \times (-2) = -6, 2 \times (-2) = -4, 1 \times (-2) = — 2, 0 х (-2) = 0$. Обратите внимание, что в каждом случае, когда мы уменьшаем первый множитель на 1, произведение увеличивается на 2. Таким образом, для согласованности следующим произведением в шаблоне должно быть $0 + 2 = 2$. Следовательно, мы имеем $(-1) \times (-2) = 2, (-2) \times (-2) = 4$, а также положительное значение для любого другого отрицательного числа, умноженное на отрицательное.

$\endgroup$

2

$\begingroup$

Я бы выбрал переворачивающее объяснение отрицательных чисел: умножение на отрицательное число переворачивает с положительного на отрицательное и с отрицательного на положительное.
Представьте, он понимает, что умножение на 1 не имеет значения, тогда все очень просто:
-1 * 1 = -1 может означать две вещи (для детей факт коммутативности умножения очевиден):
— либо сохраняет -1 как -1
— либо переворачивает +1 (положительное число) в отрицательную сторону

-1 * (-1), либо просто переворачивает обратно с отрицательной в положительную сторону.

Удачи

$\endgroup$

$\begingroup$

Если ваш сын имеет четкое представление о деньгах и знает, что такое кредитная карта, это может быть хорошим объяснением:

Представьте , что вы говорите своему сыну, что купите ему $\color{green}{\mathrm Подарочные сертификаты на {семь}}$ на сумму £$\color{green}{5}$ каждый и оплатите их кредитной картой. Объясните сыну, что теперь вы должны денег, и скажите, что это $7 \times -5=-\mathrm{£}35$. Не является частью вашего вопроса; но это покрывает случай $ \text{positive} \times \text{negative} = \text{negative}$.

Вы ужинаете со своим лучшим другом, когда приходит счет от кредитной компании. Когда ваш друг видит счет, он великодушно настаивает на том, чтобы оплатить его за вас. Теперь у вас есть подарочные купоны на £$\color{blue}{35}$, вы ничего не заплатили.

Итак, вы говорите своему сыну, что ваш лучший друг $\color{red}{\fbox{забрал}}$ семь $\color{red}{\fbox{debts}}$ из £5$ ($\color {red}{-7}\times\color{red}{-5}$), и это равняется выигрышу в £$\color{blue}{35}$.

$\endgroup$

1

$\begingroup$

Возможно, ваш ребенок познакомился со знаком минус с помощью слова напротив . Это отличный термин для использования в ваших разговорах. В самом деле, $a$ и $-a$ являются противоположностями в том смысле, что они являются аддитивными инверсиями. На числовой прямой противоположные числа зеркально отражены на расстоянии от нуля, что также обеспечивает хорошую визуальную помощь. Мы можем использовать этот термин для описания арифметических операций:

Число, противоположное трем, умноженным на пять, равно числу, противоположному числу 15. $$-3 \times 5 = -15$$

Противоположное число трижды, противоположное пяти, противоположно противоположному пятнадцати… что равно пятнадцати. $$-3 \times -5 = 15$$

Итак, мы можем использовать язык, чтобы лучше понимать отрицательные числа. А как насчет физического примера? Ну, вот симпатичный, который мне однажды рассказал друг. Немного наигранно, но думаю, что суть ясна. Вы также можете превратить это в демонстрацию.

Предположим, кубик льда снижает температуру напитка на $1$ градус.

Помещение трех кубиков льда в стакан понизит температуру на $3$ градуса, или $$ 3 \раз -1 = -3$$

Удаление двух кубиков льда повысит температуру на $2$ градуса, или $$ -2 \times -1 = 2$$

$\endgroup$

7

$\begingroup$

Проще всего объяснить, используя целые числа. Предположим, что $P$ — некоторое положительное число. Тогда $-P$ отрицательно. Теперь $-2P$ — это $P$, вычтенное из $-P$, поэтому оно по-прежнему отрицательно. Вычтите еще один $P$, и вы получите $-3P$, что по-прежнему отрицательно. Аналогично для $-4P, -5P$ и так далее. Таким образом, отрицательное время, положительное, является положительным. Та же идея для положительных времен отрицательных.

Когда дело доходит до отрицательного, умноженного на отрицательное, это немного сложнее… Но как насчет… $-P$ отрицательное, поэтому $-(-P)$ теперь положительное, переворачивая вокруг $0$. Таким образом, $-2(-P)$, добавленное к самому себе $-(-P)$, по-прежнему положительно. В общем, добавление $-(-P)$ к самому себе $Q$ раз дает $(-Q)(-P)$, что, следовательно, также положительно.

$\endgroup$

$\begingroup$

  1. Объясните определение отрицательных чисел.
  2. Укажите, что из определения $-x$ следует, что $-(-x) = x$.
  3. Объясните, что $-x = (-1)\times x$.
  4. Обратите внимание, что из (2) и (3) следует, что $(-1)\times(-1) = 1$.

$\endgroup$

4

$\begingroup$ 9{\circ}$ против часовой стрелки, чтобы умножить на $i$.

Но это уже другая история.

$\endgroup$

$\begingroup$

Символика: $$\begin{align*} -a \times -b &= (-1 \times a)\times (-1 \times b) \\ &= -1 \раз a \раз -1 \раз b \\ &= -1 \times -1 \times a \times b \\ &= (-1 \times -1) \times (a \times b) \\ &= а \ умножить на б. \end{align*}$$

Что происходит?:
Рассмотрим числовую прямую. Когда я умножаю что-то на $2$, я удваиваю расстояние от $0$. Это происходит независимо от того, является ли «что-то» положительным или отрицательным. Когда я умножаю что-то на $-2$, я удваиваю расстояние от $0$ до , а переворачиваю на другую сторону числовой прямой. Например, $-2 \times 3 : 3 \rightarrow 6 \rightarrow -6$. Если я начну с отрицательного числа, оно уже находится в отрицательной половине числовой строки и будет перевернуто на положительную половину: $-2 \times -3 : -3 \rightarrow -6 \rightarrow 6$.

То есть умножение на минус это то же самое, что два шага: умножение на вещь, как если бы она не имела минуса, затем применение знака минус. Об этом говорит символика выше: умножение на $-a$ равносильно умножению на $a$, затем на $-1$ и аналогично для $-b$. Но тогда два «переворота на другую половину числовой строки», два «$-1$», вызывают два переворота. Но два переворота что-то удаляют, а затем возвращают обратно к себе, так что два переворота на самом деле ничего не делают. Таким образом, $-1 \times -1$ равносильно ничегонеделанию или умножению на $1$. Вот почему в приведенных выше символах мы можем опустить «$-1 \times -1$» — это то же самое, что умножить на $1$.

$\endgroup$

1

$\begingroup$

Ну… это всегда имело смысл для меня (но я обнаружил, что это не имеет смысла для других)

A) Умножение — это сложение числа несколько раз.

1) Отрицательные числа — это числа, которые меньше нуля. Сокращение положительных чисел. Это античисла

Итак)i) положительное х положительное: сложите кучу положительных чисел положительное количество раз. Результат: Большой положительный прирост.

ii) положительное x отрицательное: добавьте кучу отрицательных античисел. Результатом является большое количество потенциальных отмен. Результат: отрицательный.

iii) отрицательное x положительное: возьмите кучу положительных чисел и уберите их. Результат: убыток; отрицательный.

iv) отрицательный x отрицательный: возьмите кучу античисел и уберите их. Убирая отнятие, остается вернуть вещи. Если вы аннигилируете аннигиляцию, результатом будет чистая прибыль: Результат: положительный.

$\endgroup$

$\begingroup$

Почему бы не посмотреть таблицу умножения? Давайте сделаем маленький, включая несколько отрицательных чисел. Конечно, вы можете сделать его больше, чтобы узоры были более четкими. Начнем с того, что мы уже знаем: $$\begin{массив}{|с|с|с|с|с|с|} \hline &\textbf{-2}& \textbf{-1} & \textbf{0} & \textbf{1} & \textbf{2} \\ \hline \textbf{-2} & & & & &\\ \hline \textbf{-1} & & & & & &\\ \hline \textbf{0} & & & 0 & 0 & 0\\ \hline \textbf{1} & & &0 &1 &2 \\ \hline \textbf{2} & & & 0& 2&4\\ \hline \end{массив}$$ Теперь давайте просто заметим, что третья строка (то есть первая заполненная единицей) постоянна — это просто набор нулей, поэтому мы должны расширить ее аналогичным образом. Четвертая строка, если читать справа налево, отсчитывает $2$, затем $1$, затем $0$, поэтому мы должны продолжать обратный отсчет, чтобы заполнить $-1$ и $-2$. Последняя строка отсчитывается по двое, поэтому она должна продолжаться до $-2$, а затем до $-4$. Давайте заполним это: $$\begin{массив}{|с|с|с|с|с|с|} \hline &\textbf{-2}& \textbf{-1} & \textbf{0} & \textbf{1} & \textbf{2} \\ \hline \textbf{-2} & & & & &\\ \hline \textbf{-1} & & & & & &\\ \hline \textbf{0} & 0 & 0 & 0 & 0 & 0\\ \hline \textbf{1} & -2&-1 &0 &1 &2 \\ \hline \textbf{2} &-4 & -2& 0& 2&4\\ \hline \end{массив}$$ Если мы в каждом столбце проделаем то же самое, мы сможем заполнить таблицу. Например, первый столбец считает вверх на $2$, когда мы движемся вверх — он идет $-4$, затем $-2$, затем $0$, поэтому мы должны продолжать считать таким образом до $2$, затем $4$. Если мы применим те же рассуждения к каждому столбцу, мы можем заполнить всю таблицу $$\begin{массив}{|с|с|с|с|с|с|} \hline &\textbf{-2}& \textbf{-1} & \textbf{0} & \textbf{1} & \textbf{2} \\ \hline \textbf{-2} &4& 2& 0& -2& -4\\ \hline \textbf{-1} &2& 1& 0& -1& -2\\ \hline \textbf{0} & 0 & 0 & 0 & 0 & 0\\ \hline \textbf{1} & -2&-1 &0 &1 &2 \\ \hline \textbf{2} &-4 & -2& 0& 2&4\\ \hline \end{массив}$$ И если мы проследим шаги, которые мы использовали для создания этой правильной таблицы, мы можем восстановить $(-1)\times (-1)=1$ следующим образом:

  • Во-первых, заметим, что один раз что-то оставляет эту вещь неизменной. Итак, $1\times(-1)=-1$.

  • Во-вторых, снова взглянув на таблицу, мы видим, что умножение на $(-1)$ «меняет» порядок нашего обычного счета — то есть $(-1)\times 2$ равно $-2$, тогда $( -1)\times 1$ — это еще один при $-1$ и $(-1)\times 0 =0$. Таким образом, когда мы получаем $(-1)\times (-1)$, мы должны быть на больше , чем $0$, поскольку $-1$ на один меньше , чем $0$.

Также может быть хорошо, чтобы смотрел на стол — это очень симметрично. Мы видим, например, во втором и четвертом столбцах (умножение на $1$ и $-1$) явное изменение порядка, что более или менее говорит нам о том, что на самом деле делает умножение на $-1$.

$\endgroup$

0

$\begingroup$

Ну, я думаю об этом так. У нас есть неотрицательные целые числа (0,1,2,3,4 и т. д.).

Мы вводим отрицательные числа и должны определить умножение с отрицательными числами, чтобы иметь внутреннюю согласованность.

Мы хотим сохранить свойство 0*что-либо = 0, отрицательное или положительное.

Мы также хотим сохранить дистрибутивную собственность.

Чтобы сохранить два вышеуказанных свойства, мы вынуждены определить произведение двух отрицательных чисел как положительное.

0*(-3) = 0

(5 + (-5))*(-3) = 0 (подставляю 5+ (-5) вместо нуля)

5*(-3) + (-5)*(-3) = 0 (распределительное свойство)

добавить 5*3 к обеим сторонам. 5*3 отменяется с помощью 5*(-3)

(-5)*(-3) = 5*3

$\endgroup$

Как умножать отрицательные числа

Как умножать отрицательные числа — ACT Math

—>

  • Войти
  • Биографии репетитора
  • Подготовка к тесту
    СРЕДНЯЯ ШКОЛА
    • ACT Репетиторство
    • SAT Репетиторство
    • Репетиторство PSAT
    • ASPIRE Репетиторство
    • ШСАТ Репетиторство
    • Репетиторство STAAR
    ВЫСШАЯ ШКОЛА
    • Репетиторство MCAT
    • GRE Репетиторство
    • Репетиторство по программе LSAT
    • Репетиторство по GMAT
    К-8
    • Репетиторство AIMS
    • Репетиторство по HSPT
    • Репетиторство ISEE
    • Репетиторство по ISAT
    • Репетиторство по SSAT
    • Репетиторство STAAR
    Поиск 50+ тестов
  • Академическое обучение
    репетиторство по математике
    • алгебра
    • Исчисление
    • Элементарная математика
    • Геометрия
    • Предварительное исчисление
    • Статистика
    • Тригонометрия
    репетиторство по естественным наукам
    • Анатомия
    • Биология
    • Химия
    • Физика
    • Физиология
    иностранные языки
    • французский
    • немецкий
    • Латинский
    • Китайский мандарин
    • Испанский
    начальное обучение
    • Чтение
    • Акустика
    • Элементарная математика
    прочее
    • Бухгалтерский учет
    • Информатика
    • Экономика
    • Английский
    • Финансы
    • История
    • Письмо
    • Лето
    Поиск по 350+ темам
  • О
    • Обзор видео
    • Процесс выбора наставника
    • Онлайн-репетиторство
    • Мобильное обучение
    • Мгновенное обучение
    • Как мы работаем
    • Наша гарантия
    • Влияние репетиторства
    • Обзоры и отзывы
    • Освещение в СМИ
    • О преподавателях университета

Мы открыты в субботу и воскресенье!

Звоните прямо сейчас, чтобы записаться на обучение:

(888) 888-0446

Все математические ресурсы ACT

14 Диагностические тесты 767 практических тестов Вопрос дня Карточки Learn by Concept

ACT Math Help » Арифметика » Целые числа » Отрицательные числа » Как умножить отрицательные числа

Если a = –2 и b = –3, затем оцените 3 + B 2

Возможные ответы:

5

17

8

1

Правильный ответ:

1

Объяснение:

При умножении отрицательных чисел мы получаем отрицательный ответ, если умножается нечетное количество отрицательных чисел. Мы получаем положительный ответ, если умножаем четное число отрицательных чисел.

a 3 + b 2 становится (–2) 3 + (–3) 2 что равно –8 + 9 = 1

0 Сообщить об ошибке

Оценка:

–3 * –7

Возможные ответы:

–10

10

–21

21

4

Правильный Ответ:

21

9 . Объяснение:

Умножение отрицательного числа на другое отрицательное число дает положительное произведение.

Сообщить об ошибке

Оценить.

Возможные ответы:

Правильный ответ:

Объяснение:

 

Умножение отрицательного и положительного числа дает отрицательное произведение:

      

Сообщить об ошибке

Решить.

Возможные ответы:

Правильный ответ:

Объяснение:

Отрицательное число, умноженное на положительное число, всегда будет отрицательным.

Отчет о ошибке

Оценка 3x 3 + x 2 , если x = 2

. 14

Правильный ответ:

20

Объяснение:

При умножении отрицательного числа нечетное число раз ответ будет отрицательным. При умножении отрицательного числа четное число раз ответ положительный. Также применяется порядок операций: Скобки, Экспоненты, Умножение и деление, Сложение и вычитание, слева направо. Мнемоника для запоминания порядка действий: «Пожалуйста, извините, моя дорогая тетя Салли».

3 ( 2) 3 + ( 2) 2

= 3 ( 8) + (4)

= 24 + 4

44444444444 = 554 + 4

4444444444 = 24 + 4

44444444 = 24 + 4

844444444444 = 24 + 4). 20

Отчет о ошибке

Упростить следующее выражение: (–4) (2) ( — 1) ( — 3)

Возможные ответы:

–24

–16

24

12

Правильный ответ:

–24

Объяснение:

Сначала мы умножаем –4 на 2. Перемножение отрицательного и положительного числа дает отрицательное число, поэтому (–4)(2) = –8. Отрицательное, умноженное на отрицательное, является положительным, поэтому (–8)(–1) = 8. (8)(–3) = –24.

Сообщить об ошибке

Оцените следующее:

Возможные ответы:

Правильный ответ:

Объяснение:

Умножение двух нечетных чисел всегда дает четное число. Просто умножьте два, как если бы они были четными. Таким образом

Сообщить об ошибке

Уведомление об авторских правах

Посмотреть репетиторов ACT по математике

Наташа Мохамед
Сертифицированный репетитор

Малабарский христианский колледж, бакалавр наук, математика. Учебный колледж Фарука, бакалавр педагогики, преподаватель математики…

View ACT Math Tutors

Josiah
Сертифицированный репетитор

Университет Миссури-Сент-Луис, бакалавр наук, мульти-/междисциплинарные исследования, общие.

Посмотреть ACT Репетиторы по математике

Woo
Сертифицированный преподаватель

Университет Сунгюнкван в Сеуле, Корея, бакалавр технических наук, металлургическая инженерия.

Все математические ресурсы ACT

14 диагностических тестов 767 практических тестов Вопрос дня Карточки Учитесь по концепции

Ресургоголик: умножение негатива

«Отрицательное, умноженное на отрицательное, становится положительным». Это трудно объяснить. Мы все учили его в школе и практиковали до беглой речи, но только после того, как нас спросят почему это работает, что мы остановимся и подумаем об этом.

Числовые линии и визуализации очень полезны при обучении сложению и вычитанию отрицательных чисел. А вот с умножением и делением не все так однозначно.

Давайте рассмотрим несколько подходов и ресурсов.

1 . Pattern Spotting
Нарисуйте стандартную таблицу умножения и расширьте ее, включив в нее отрицательные числа. Это простая схема, которую все учащиеся должны уметь заметить и продолжить. Предложите учащимся сделать это, используя упражнение Колина Фостера на странице 5 его главы «Отрицательные числа».

2. Сетки умножения
Возьмите два двузначных числа и умножьте их вместе, используя умножение сетки. Для простоты возьмем 12 x 11:

Здесь мы записали 12 как 10 + 2 и 11 как 10 + 1. Но было бы также хорошо, если бы мы выражали эти числа по-другому. Вместо этого давайте запишем 12 как 15 — 3 и 11 как 15 — 4. Мы должны получить тот же ответ:

Это работает, только если -3 x -4 = 12.

Обратите внимание, что это объяснение требует, чтобы учащиеся сначала поняли, что положительный х отрицательный = отрицательный. Это относительно просто объяснить с точки зрения многократного добавления.

3. Доказательство
Вот доказательство, ясное и доступное для нас, опытных математиков. Я не уверен, насколько это доступно для учащихся 7 класса, но оно того стоит.

a и b положительные
а + (-а) = 0
[а +(-а)]•b = 0•b
a•b + (-a)•b = 0
a•b положительный. Следовательно, (-a)•b отрицательно

б + (-б) = 0
(-а)•[б + (-б)] = (-а)•0
(-а)•б + (-а)•(-б) = 0
Поскольку (-a)•b отрицательно, мы заключаем, что (-a)•(-b) положительно.

Возможно, вместо формального доказательства начните с числового примера.

3 + (-3) = 0
Умножить все на -4
3(-4) + (-3)(-4) = 0(-4)  
-12 + (-3)(-4)  = 0  
(-3)(-4) должно равняться 12, чтобы это утверждение было верным.  

Дополнительная литература
Рекомендуется прочитать о теме, прежде чем преподавать ее, даже относительно простые темы, которые вы преподавали много раз ранее. Вот несколько полезных ссылок:

  • На страницах часто задаваемых вопросов на Math Forum всегда можно найти интересные ответы: Почему отрицательное число, умноженное на отрицательное, является положительным?
  • Мне нравятся эссе Джеймса Тантона из учебного плана: Почему отрицательное время становится отрицательным положительным?
  • Я был вдохновлен на написание этого поста после просмотра отличного видео «Негатив, умноженный на негатив, это. ..» от Mathologer (мой новый любимый канал на YouTube!).
  • Стоит прочитать «Историю отрицательных чисел» от NRICH.

Мне нравится этот клип из Stand and Deliver:

Здесь важна формулировка. «Отрицательное, умноженное на отрицательное, равно положительному» явно предпочтительнее, чем «два минуса дают плюс». Последнее сбивает с толку и может привести к неправильным представлениям. Пример распространенной ошибки показан ниже (взято с mathmistakes.org через Nix the Tricks).

Задачи и ресурсы
Вот несколько рекомендаций по ресурсам по этой теме:

  • Мне очень нравится задание Дона Стюарда «Направленные пробелы в числах» — оно хорошо работает с любой группой года
  • В MathsPad есть множество головоломок с отрицательными числами, включая арифмагоны (некоторые из этих ресурсов доступны только подписчикам)
  • В главе
  • CIMT об отрицательных числах есть задания по умножению отрицательных чисел.

Колин Фостер предлагает вам попросить учеников составить десять умножений и десяти делений, каждое из которых дает ответ –8 (например, –2 × –2 × –2 или –1 × 8 и т. д.).

Возведение в квадрат и куб (и т.д.) отрицательных значений стоит обсудить — учащиеся должны заметить, что четная степень дает положительное значение (например, каково значение (-1) 100 ?).

Возможно, стоит также изучить поведение калькулятора (например, некоторые калькуляторы требуют квадратных скобок при возведении в квадрат отрицательного значения). Важно, чтобы учащиеся знали, как правильно пользоваться калькулятором. Для этого есть отличный ресурс от MathsPad — Using a Calculator: Odd One Out.

К этой теме возвращаются в последующие годы, когда учащиеся практикуют замещение. Например, если a = 3, b = -2 и c = -5, найдите значения: abc; до н.э. 2 ; (бк) 2 ; а 2 б 3 и так далее. Эта головоломка с заменой от mathsteaching. wordpress.com становится довольно сложной.

Сообщите мне, если вы используете интересный метод или ресурс для обучения умножению отрицательных чисел.

«Минус, умноженный на минус, дает плюс,
Причину этого нам не нужно обсуждать»
— Огден Нэш

Умножить или добавить сначала? Преподавание порядка действий Правила

Вернуться к форме

Математика

Фигурный посох

9 мин Чтение

Когда учащиеся 3-х классов и старше сначала учатся складывать, вычитать, умножать, делить и работать с основными числовыми выражениями, они начинают с выполнения операций над двумя числами. Но что происходит, когда выражение требует нескольких операций? Например, вы сначала складываете или умножаете? А умножить или разделить? В этой статье объясняется, что такое порядок операций, и приводятся примеры, которые вы также можете использовать со студентами. Он также содержит два урока, которые помогут вам представить и развить эту концепцию.

Стандартный ключ:

  • Выполнять арифметические операции, включая сложение, вычитание, умножение и деление в обычном порядке, независимо от того, есть скобки или нет. (Класс 3)

Порядок операций является примером очень процедурной математики. Легко запутаться, потому что это не столько концепция, которую вы осваиваете, сколько список правил, которые вы должны запомнить. Но не обманывайте себя, думая, что процедурные навыки не могут быть глубокими! В нем могут быть представлены сложные задачи, подходящие для старших школьников и созревшие для обсуждения в классе:

  • Изменяется ли правило слева направо, когда умножение подразумевается, а не прописывается? (Например, \(3g\) или \(8(12)\) вместо \(3 \times g\) или \(8 \cdot 12\).)
  • Где факториал попадает в порядок операций ?
  • Что произойдет, если вы возвели один показатель степени в другой показатель степени, но скобок нет? (Обратите внимание, что этот урок не включает показатели, хотя, если учащиеся готовы, вы можете расширить свой урок, включив их. )

Что важнее в порядке операций?

Со временем математики пришли к соглашению о наборе правил, называемых порядком операций , чтобы определить, какую операцию выполнять первой. Когда выражение включает только четыре основные операции, действуют следующие правила:

  1. Умножение и деление слева направо.
  2. Сложение и вычитание слева направо.

При упрощении выражения, такого как \(12 \div 4 + 5 \times 3 — 6\), сначала вычислите \(12 \div 4\), поскольку порядок операций требует сначала вычисления любого умножения и деления (в зависимости от того, что произойдет сначала) слева направо перед оценкой сложения или вычитания. В данном случае это означает, что сначала нужно вычислить \(12 \div 4\), а затем \(5 \times 3\). Как только все умножение и деление завершены, продолжайте складывать или вычитать (в зависимости от того, что наступит раньше) слева направо. Шаги показаны ниже.

(− a ) × (− b ) = ( a × (−1)) × (− b ) (Следствие 3)
(− a ) × (− b 9 (05 × ) 4 = −1) × (− b )) (аксиома 6)
(− a ) × (− b ) = a × (−01 7 955 b 9 ) (Следствие 3)
(− a ) × (− b ) = a × b (Следствие 1)
\(12 \дел 4 + 5 \умножить на 3 — 6\)
\(3 + 5 \умножить на 3 — 6\) Потому что \(12 \дел 4 = 3\)
\(3 + 15 — 6\) Потому что \(5 \х3 = 15\)
\(18 — 6\) Потому что \(3 + 15 = 18\)
\(12\) Потому что \(18 — 6 = 12\)

Рассмотрим в качестве примера другое выражение:

\(6 + 4 \times 70186
\(6 + 28 — 3\) Потому что \(4 \times 7 = 28\), что выполняется первым, потому что сначала вычисляются умножение и деление.
\(34 — 3\) Потому что \(6 + 28 = 34\)
\(31\) Потому что \(34 — 3 = 31\) 9018 6 мы могли бы захотеть убедиться, что сложение или вычитание выполняется в первую очередь. Символы группировки , такие как круглые скобки \(( )\), скобок \([ ]\) или скобок \(\{ \}\) позволяют нам определить порядок выполнения конкретных операций.

Порядок операций требует, чтобы операции внутри группирующих символов выполнялись до операций вне их. Например, предположим, что выражение 6 + 4 заключено в круглые скобки:

\((6 + 4) \times 7 — 3\)
\(10 \times 7 — 3\) Потому что \(6 + 4 = 10\), что делается первым, потому что оно заключено в круглые скобки.
\(70 — 3\) Потому что \(10 \times 7 = 70\), и скобок больше нет.
\(67\) Потому что \(70 — 3 = 67\)

Обратите внимание, что выражение имеет совершенно другое значение! Что, если вместо этого мы заключим в скобки \(7 — 3\)?

\(6 + 4 \умножить на (7 — 3)\)
\(6 + 4 \умножить на 4\) На этот раз \(7 — 3\) в скобках, поэтому мы делаем это в первую очередь.
\(6 + 16\) Поскольку \(4 \times 4 = 16\), и когда не осталось скобок, мы продолжаем умножение перед сложением.
\(22\) Потому что \(6 + 16 = 22\)

Этот набор скобок дает еще один ответ. Итак, когда задействованы круглые скобки, правила порядка операций следующие:

  1. Выполнять операции в круглых скобках или группировать символы.
  2. Умножение и деление слева направо.
  3. Сложение и вычитание слева направо.

Знакомство с концепцией: порядок операций

Прежде чем ваши учащиеся будут использовать скобки в математике, они должны четко понимать порядок операций без скобок. Начните с повторения правил сложения и умножения в порядке выполнения операций, а затем покажите учащимся, как круглые скобки могут повлиять на этот порядок.

Материалы: Белая доска или способ писать для класса публично

Необходимые навыки и концепции: Учащиеся должны уметь оценивать и обсуждать выражения сложения, вычитания, умножения и деления.

  • Спросить : Какую операцию выполнить первой в выражении \(5 \times 7 + 3\) ? Почему?

    Запишите выражение публично. Если студенты не согласны, попросите их объяснить, не говоря им, правы они или нет. При необходимости напомните им, что по порядку операций умножение и деление предшествуют сложению и вычитанию.

  • Спросите : Каково значение этого выражения?

    Попросите учащихся оценить выражение. \(5 \times 7 = 35\), поэтому выражение становится \(35 + 3\), что равно \(38\).

  • Спросите : Что произойдет, если я поменяю местами символы сложения и умножения? Какое значение я получу?

    Перепишите выражение как \(5 + 7 \умножить на 3\) и выполните вычисление. \(7 \times 3 = 21\), поэтому выражение становится \(5 + 21\), что равно \(26\).

  • Спросите : Получили ли мы другие значения при изменении операций?

    Этот результат, вероятно, не удивит ваших учеников. Скорее всего, они знают, что выполнение разных операций над одними и теми же числами даст разные значения. Если позволяет время и учащиеся готовы, предложите им найти выражение, в котором перестановка символов сложения и умножения, как вы сделали, дает одно и то же значение. Если кто-то из учащихся преуспеет, попросите их показать, как они получили выражения. Обратите внимание, что это возможно только тогда, когда среднее число равно 1 (например, \(5 \times 1 + 3\) или \(5 + 1 \times 3\)) или внешние числа равны (например, \(3 \times 7). + 3\) или \(3 + 7 \умножить на 3\)).

  • Спросите : Что делать, если я хочу оставить символы умножения и сложения на одном месте (\(5 \times 7 + 3\)) , но выполнить \(7 + 3\) сначала ? Как вы думаете, как я мог это сделать?

    Кратко обсудите вопрос, затем напишите на доске \(5 \раз (7 + 3)\). Обратите внимание на скобки.

  • Скажем : Мы называем эти символы скобками. Если в выражении есть скобки, сначала сделайте то, что внутри скобок.
  • Спросите : Что находится в скобках в выражении \(5 \times (7 + 3)\) ?

    Убедитесь, что учащиеся правильно понимают, что число \(7 + 3\) находится внутри скобок и что оно должно оцениваться перед вычислением с помощью \(5\).

  • Скажем : Теперь давайте закончим вычисление значения. (Значение равно \(5 \times 10\) или \(50\).) Это то же самое значение, которое мы получили раньше?

    Помогите учащимся заметить, что значение не совпадает ни с исходным выражением, ни с выражением с переключенными символами операций.

Это подходящий момент для обсуждения математической практики с учетом точности . В математике очень важно, чтобы мы преднамеренно писали математические выражения и делали математические утверждения. Небольшие перепутывания с математическими правилами операций или скобками могут привести к радикальным изменениям! Представьте себе неправильное вычисление выражения, например, при расчете дозировки или стоимости лекарства.

Дайте учащимся еще несколько примеров, показывающих выражение со скобками и без них. Попросите студентов-добровольцев оценить выражения и сравнить их значения. Когда учащиеся приходят к разным значениям, не говорите им, правы они или нет. Вместо этого предложите им найти сходства и различия в своих стратегиях и направьте обсуждение так, чтобы учащиеся увидели, какая стратегия соответствует правилам порядка действий.

Разработка концепции: Порядок действий

Материалы: Белая доска или способ записи в классе публично

Предварительные навыки и понятия: Учащиеся должны быть знакомы с порядком действий и чувствовать себя готовыми применять его на практике.

Продолжая обучать своих студентов работе со скобками, убедитесь, что они не всегда изменяют значение выражения, хотя часто изменяют.

  • Спросить : Какую операцию выполнить первой в выражении \(3 + 5 \times 8\) и почему?

    Запишите выражение публично. Убедитесь, что учащиеся ясно понимают, что порядок операций требует, чтобы они выполняли умножение перед сложением.

  • Спросите : Что произойдет, если я хочу добавить 3 и 5 до умножения на 8?

    Позвольте учащимся обсудить идеи о том, как изменить порядок операций. Не говорите ученикам, что они правы, а что нет. Вместо этого поощряйте математический дискурс и сравнивайте разные мнения, чтобы исправить неправильные представления. Обратите внимание, что вариантов ответов может быть много! Например, в задаче может быть явно указано «сначала добавьте 3 и 5», или исторически существовали другие способы группировки, такие как использование горизонтальных черт над выражением. Если они не упоминают скобки, напомните им, что вы делали на первом уроке.

  • Скажем : Заключив скобки вокруг \(3 + 5\) , мы говорим, что мы должны сначала сложить 3 и 5, а затем умножить на 8. Сегодня мы собираемся попрактиковаться в нахождении значения выражений с и без скобок и посмотрите, какое значение имеют скобки.
  • Напишите следующие три выражения публично, чтобы все учащиеся могли их увидеть.
    • \(3 + 6 \умножить на 2\)
    • \((3 + 6) \умножить на 2\)
    • \(3 + (6 \умножить на 2)\)
  • Произнесите : Вычислите все три выражения.

    Дайте учащимся время закончить вычисления. Затем пусть студенты-добровольцы сообщат о том, что они нашли.

  • Спросите : Вы получили одинаковое значение для всех трех выражений? Почему или почему нет?

    Учащиеся должны заметить, что выражения 1 и 3 дают одно и то же значение, а выражение 2 отличается. Обсудите, что выражение 2 требует сложения перед умножением, а выражения 1 и 3 требуют умножения перед сложением. Цель состоит в том, чтобы учащиеся увидели, что использование скобок иногда меняет значение выражения, а иногда нет.

  • Напишите следующие два выражения публично, чтобы все учащиеся могли их увидеть.
    • \((8 \дел 4) — 2\)
    • \(8 \дел (4 — 2)\)
  • Произнесите: Вычислите оба выражения.

    Дайте учащимся время закончить вычисления. Затем пусть студенты-добровольцы сообщат о том, что они нашли.

  • Спросите : Являются ли значения этих выражений одинаковыми? Почему или почему нет?

    Еще раз учащиеся должны увидеть значение использования скобок.

  • Скажите: Теперь мы попробуем задание со многими возможными решениями. Ваша цель — найти выражение, в котором можно перемещать скобки без изменения значения. Проблема заключается в том, что круглые скобки должны быть около сложения или вычитания .

    Пройдите пример. Покажите, как в двух приведенных ниже выражениях скобки окружают выражение сложения, и когда они перемещаются, значение выражения остается прежним: 7.

    • \((3 + 4) \умножить на 1\)
    • \(3 + (4 \умножить на 1)\)
  • Если это возможно, попросите учащихся работать в парах, чтобы создать дополнительные примеры. Учащимся, которые застряли, попросите их заменить 3 и/или 4 в приведенных выше выражениях.
  • Спросите : Как вы создавали выражения, которые позволяли вам «двигать» скобки? С какими проблемами вы столкнулись?

    Организуйте обсуждение различных выражений, сделанных учащимися. Предложите учащимся сравнить сходства и различия как в выражениях, которые они сделали, так и в стратегиях, которые они использовали для их выражения.

Подведение итогов и советы по оценке

Важно, чтобы учащиеся могли запомнить правила порядка операций как со скобками, так и без них. Избегайте давать рабочие листы механического обучения. Вместо этого ищите математические задачи, которые естественным образом приводят к выражениям, которые необходимо вычислить, например, подстановка значений в формулу, и попросите учащихся попрактиковаться в порядке выполнения операций в контексте других задач.

***

Хотите повысить уверенность учащихся в математике, помимо практики математических правил порядка операций? Исследуйте HMH Into Math , наше базовое математическое решение K–8.

Математика 3-5 классы 6-8 классы Занятия и уроки

Дополнительная литература